Метод гаусса метод исключения: Решение систем линейных уравнений методом Гаусса

Содержание

НОУ ИНТУИТ | Лекция | Умножение разреженных матриц

Аннотация: В лекции рассматриваются типовые алгоритмы, применяемые в работе с плотными и разреженными матрицами. Рассказывается о некоторых форматах хранения этих матриц. Рассматриваются основные свойства матриц, методы решения СЛАУ, прямые и итерационные методы.

Цель лекции: Основной целью лекции является применение полученные знания о разработке и отладке параллельных программ в реализации численных алгоритмов для работы с матрицами.

Прямые методы решения СЛАУ

Методы решения систем линейных алгебраических уравнений (СЛАУ) относятся к численным методам алгебры. При формальном подходе решение подобных задач не встречает затруднений: решение системы можно найти, раскрыв определители в формуле Крамера. Однако при непосредственном раскрытии определителей решение системы с n неизвестными требует арифметических операций; уже при n порядка 20 такое число операций недоступно для современных компьютеров. При сколько-нибудь больших n применение методов с таким порядком числа операций будет невозможно и в обозримом будущем. Другой причиной, по которой этот классический способ неприменим даже при малых n, является сильное влияние на окончательный результат округлений при вычислениях.

Методы решения алгебраических задач можно разделить на точные и итерационные. Классы задач, для решения которых обычно применяют методы этих групп, можно условно назвать соответственно классами задач со средним и большим числом неизвестных. Изменение объема и структуры памяти вычислительных систем, увеличение их быстродействия и развитие численных методов приводят к смещению границ применения методов в сторону систем более высоких порядков.

Например, в 80-х годах прошлого века точные методы применялись для решения систем до порядка 104, итерационные – до порядка 107, в 90-х – до порядков 105 и 108 соответственно. Современные суперкомпьютеры способны использовать точные методы при решении еще больших систем.

Мы будем рассматривать систему из

n линейных алгебраических уравнений вида

( 7.1)

В матричном виде система может быть представлена как

( 7.2)

где есть вещественная матрица размера ; b и x – вектора из n элементов.

Под задачей решения системы линейных уравнений для заданных матрицы А и вектора b мы будем считать нахождение значения вектора неизвестных x, при котором выполняются все уравнения системы.

Метод исключения Гаусса

В первую очередь рассмотрим алгоритмыы, предназначенные для решения системы

( 7.3)

с произвольной квадратной матрицей А. Основой для всех них служит широко известный метод последовательного исключения неизвестных, или же метод Гаусса.

Метод Гаусса основывается на возможности выполнения преобразований линейных уравнений, которые не меняют при этом решение рассматриваемой системы (такие преобразования носят наименование эквивалентных). К числу таких преобразований относятся:

  • умножение любого из уравнений на ненулевую константу,
  • перестановка уравнений,
  • прибавление к уравнению любого другого уравнения системы.

Метод Гаусса включает последовательное выполнение двух этапов. На первом этапе, который называется прямой ход, исходная система линейных уравнений при помощи последовательного исключения неизвестных приводится к верхнему треугольному виду. При выполнении обратного хода (второй этап алгоритма) осуществляется определение значений неизвестных.

Последовательный алгоритм

Прямой ход состоит в последовательном исключении неизвестных в уравнениях решаемой системы линейных уравнений.

На итерации i, метода производится исключение неизвестной i для всех уравнений с номерами k, больших i(т.е.) Для этого из этих уравнений осуществляется вычитание строки i, умноженной на константу с тем, чтобы результирующий коэффициент при неизвестной в строках оказался нулевым – все необходимые вычисления могут быть определены при помощи соотношений:

( 7.4)

где – множители Гаусса.

В итоге приходим к системе с верхней треугольной матрицей

При выполнении прямого хода метода Гаусса строка, которая используется для исключения неизвестных, носит наименование ведущей, а диагональный элемент ведущей строки называется ведущим элементом. Как можно заметить, выполнение вычислений является возможным только, если ведущий элемент имеет ненулевое значение. Более того, если ведущий элемент

имеет малое значение, то деление и умножение строк на этот элемент может приводить к накоплению вычислительной погрешности и вычислительной неустойчивости алгоритма.

Избежать подобной проблемы можно, если при выполнении каждой очередной итерации прямого хода метода Гаусса определить коэффициент с максимальным значением по абсолютной величине в столбце, соответствующем исключаемой неизвестной, т.е.

и выбрать в качестве ведущей строку, в которой этот коэффициент располагается (данная схема выбора ведущего значения носит наименование метода главных элементов).

Обратный ход алгоритма состоит в следующем. После приведения матрицы коэффициентов к верхнему треугольному виду становится возможным определение значений неизвестных. Из последнего уравнения преобразованной системы может быть вычислено значение переменной

, после этого из предпоследнего уравнения становится возможным определение переменной и т.д. В общем виде выполняемые вычисления при обратном ходе метода Гаусса могут быть представлены при помощи соотношений:

Оценим трудоемкость метода Гаусса. При выполнении прямого хода число операций составит

Для выполнения обратного хода потребуется

Таким образом, общее время выполнения метода Гаусса при больших n можно оценить как

где время выполнения одной операции.

Параллельный алгоритм

При внимательном рассмотрении метода Гаусса можно заметить, что все вычисления сводятся к однотипным вычислительным операциям над строками матрицы коэффициентов системы линейных уравнений. Как результат, в основу параллельной реализации алгоритма Гаусса может быть положен принцип распараллеливания по данным. В качестве базовой подзадачи можно принять тогда все вычисления, связанные с обработкой одной строки матрицы A и соответствующего элемента вектора b. Рассмотрим общую схему параллельных вычислений и возникающие при этом информационные зависимости между базовыми подзадачами.

Для выполнения прямого хода метода Гаусса необходимо осуществить итерацию по исключению неизвестных для преобразования матрицы коэффициентов A к верхнему треугольному виду. Выполнение итерации i, , прямого хода метода Гаусса включает ряд последовательных действий. Прежде всего, в самом начале итерации необходимо выбрать ведущую строку, которая при использовании метода главных элементов определяется поиском строки с наибольшим по абсолютной величине значением среди элементов столбца i, соответствующего исключаемой переменной . Зная ведущую строку, подзадачи выполняют вычитание строк, обеспечивая тем самым исключение соответствующей неизвестной .

При выполнении обратного хода метода Гаусса подзадачи выполняют необходимые вычисления для нахождения значения неизвестных. Как только какая-либо подзадача i, , определяет значение своей переменной , это значение должно быть использовано всеми подзадачам с номерами k, : подзадачи подставляют полученное значение новой неизвестной и выполняют корректировку значений для элементов вектора b.

Выделенные базовые подзадачи характеризуются одинаковой вычислительной трудоемкостью. Однако размер матрицы, описывающей систему линейных уравнений, является существенно большим, чем число потоков в программе (т.е.,), и базовые подзадачи можно укрупнить, объединив в рамках одной подзадачи несколько строк матрицы. При этом применение последовательной схемы разделения данных для параллельного решения систем линейных уравнений приведет к неравномерной вычислительной нагрузке между потоками: по мере исключения (на прямом ходе) или определения (на обратном ходе) неизвестных в методе Гаусса для большей части потоков все необходимые вычисления будут завершены и они окажутся простаивающими. Возможное решение проблемы балансировки вычислений может состоять в использовании ленточной циклической схемы для распределения данных между укрупненными подзадачами. В этом случае матрица

A делится на наборы (полосы) строк вида (см. рис. 7.1).


Рис. 7.1. Ленточная схема

Сопоставив схему разделения данных и порядок выполнения вычислений в методе Гаусса, можно отметить, что использование циклического способа формирования полос позволяет обеспечить лучшую балансировку вычислительной нагрузки между подзадачами.

Итак, проведя анализ последовательного варианта алгоритма Гаусса, можно заключить, что распараллеливание возможно для следующих вычислительных процедур:

  • поиск ведущей строки,
  • вычитание ведущей строки из всех строк, подлежащих обработке,
  • выполнение обратного хода.

Оценим трудоемкость рассмотренного параллельного варианта метода Гаусса. Пусть n есть порядок решаемой системы линейных уравнений, а p, , обозначает число потоков. При разработке параллельного алгоритма все вычислительные операции, выполняемые алгоритмом Гаусса, были распределены между потоками параллельной программы. Следовательно, время, необходимое для выполнения вычислений на этапе прямого хода, можно оценить как

Подставив выражение получим, что время выполнения вычислений для параллельного варианта метода Гаусса описывается выражением:

Теперь можно оценить величину накладных расходов, обусловленных организацией и закрытием параллельных секций. Пусть – время, необходимое на организацию и закрытие параллельной секции. Параллельная секция создается при каждом выборе ведущей строки, при выполнении вычитания ведущей строки из остальных строк линейной системы, подлежащих обработке, а также при выполнении каждой итерации обратного хода метода Гаусса. Таким образом, общее число параллельных секций составляет .

Сводя воедино все полученные оценки можно заключить, что время выполнения параллельного метода Гаусса описывается соотношением

( 7.5)
Связь метода Гаусса и LU-разложения

LU-разложение – представление матрицы A в виде

( 7.6)

где L – нижняя треугольная матрица с диагональными элементами, равными единице, а U – верхняя треугольная матрица с ненулевыми диагональными элементами. LU-разложение также называют LU-факторизацией. Известно [4], что LU-разложение существует и единственно, если главные миноры матрицы A отличны от нуля.

Алгоритм LU-разложения тесно связан с методом исключения Гаусса. В самом деле, пусть мы решаем систему уравнений вида (7.2). Непосредственно проверяется, что преобразования k-го шага метода Гаусса равносильны домножению системы (7.2) слева на матрицу

где – множители Гаусса из (7.4). Как было рассмотрено в п. 7.1.1, прямой ход метода Гаусса преобразует исходную систему уравнений к виду

с верхней треугольной матрицей U. Зная матрицы , можно записать матрицу U и вектор c как

Обозначим Можно непосредственно проверить, что

Отсюда получаем .

Таким образом, матрицу L можно получить как нижнюю треугольную матрицу коэффициентов Гаусса, а матрицу U – как верхнюю треугольную матрицу, получаемую в результате работы метода Гаусса. При этом очевидно, что трудоемкость получения LU-факторизации будет такой же-.

Рассмотренный нами алгоритм LU-факторизации реализован с помощью исключения по столбцу. Следует отметить, что можно сформулировать аналогичный алгоритм, основанный на исключении по строке. В самом деле, основная идея алгоритма с помощью исключения по столбцу заключается в том, что на i-й итерации ведущая строка с подходящими множителями вычитается из строк, лежащих ниже, чтобы занулить все элементы матрицы, расположенные в i-м столбце ниже диагонали. Между тем возможно и другое: на каждой i-й итерации можно вычитать из i-й строки все строки, расположенные выше, умноженные на подходящие коэффициенты, так, чтобы занулить все элементы i-й строки левее диагонали. При этом элементы матрицы L ниже главной диагонали и элементы матрицы U на главной диагонали и выше нее можно вычислять на месте матрицы А. Как и в случае исключения по столбцу, приведенная схема требует проведения операций

Рассмотрим теперь еще один способ LU-факторизаци, называемый компактной схемой. Пусть матрица допускает LU-разложение (7.6), где

т.е. при а при . Из соотношения (7.6) следует, что

Преобразуем эту сумму двумя способами:

Отсюда находим

Оценка числа операций данного алгоритма LUфакторизаци также составляет

Если разложение (7.6) получено, то решение системы (7.2) сводится к последовательному решению двух систем уравнений с треугольными матрицами (обратный ход)

( 7.7)

Обратный ход требует операций.

Как следует из приведенных оценок, вычислительная сложность метода исключения Гаусса и метода LU-разложения одинакова. Однако если необходимо решить несколько систем с одинаковыми матрицами коэффициентов, но различными векторами свободных членов (правая часть СЛАУ), то метод LU-разложения окажется предпочтительным, так как в этом случае нет необходимости производить разложение матрицы коэффициентов многократно. Достаточно лишь сохранить полученные треугольные матрицы в памяти и, подставляя различные вектора свободных членов, получать решения методами прямой и обратной подстановки. Это позволит значительно сократить объем вычислений по сравнению с методом Гаусса.

%d0%bc%d0%b5%d1%82%d0%be%d0%b4%20%d0%b3%d0%b0%d1%83%d1%81%d1%81%d0%b0 — со всех языков на все языки

Все языкиАбхазскийАдыгейскийАфрикаансАйнский языкАканАлтайскийАрагонскийАрабскийАстурийскийАймараАзербайджанскийБашкирскийБагобоБелорусскийБолгарскийТибетскийБурятскийКаталанскийЧеченскийШорскийЧерокиШайенскогоКриЧешскийКрымскотатарскийЦерковнославянский (Старославянский)ЧувашскийВаллийскийДатскийНемецкийДолганскийГреческийАнглийскийЭсперантоИспанскийЭстонскийБаскскийЭвенкийскийПерсидскийФинскийФарерскийФранцузскийИрландскийГэльскийГуараниКлингонскийЭльзасскийИвритХиндиХорватскийВерхнелужицкийГаитянскийВенгерскийАрмянскийИндонезийскийИнупиакИнгушскийИсландскийИтальянскийЯпонскийГрузинскийКарачаевскийЧеркесскийКазахскийКхмерскийКорейскийКумыкскийКурдскийКомиКиргизскийЛатинскийЛюксембургскийСефардскийЛингалаЛитовскийЛатышскийМаньчжурскийМикенскийМокшанскийМаориМарийскийМакедонскийКомиМонгольскийМалайскийМайяЭрзянскийНидерландскийНорвежскийНауатльОрокскийНогайскийОсетинскийОсманскийПенджабскийПалиПольскийПапьяментоДревнерусский языкПортугальскийКечуаКвеньяРумынский, МолдавскийАрумынскийРусскийСанскритСеверносаамскийЯкутскийСловацкийСловенскийАлбанскийСербскийШведскийСуахилиШумерскийСилезскийТофаларскийТаджикскийТайскийТуркменскийТагальскийТурецкийТатарскийТувинскийТвиУдмурдскийУйгурскийУкраинскийУрдуУрумскийУзбекскийВьетнамскийВепсскийВарайскийЮпийскийИдишЙорубаКитайский

 

Все языкиАбхазскийАдыгейскийАфрикаансАйнский языкАлтайскийАрабскийАварскийАймараАзербайджанскийБашкирскийБелорусскийБолгарскийКаталанскийЧеченскийЧаморроШорскийЧерокиЧешскийКрымскотатарскийЦерковнославянский (Старославянский)ЧувашскийДатскийНемецкийГреческийАнглийскийЭсперантоИспанскийЭстонскийБаскскийЭвенкийскийПерсидскийФинскийФарерскийФранцузскийИрландскийГалисийскийКлингонскийЭльзасскийИвритХиндиХорватскийГаитянскийВенгерскийАрмянскийИндонезийскийИнгушскийИсландскийИтальянскийИжорскийЯпонскийЛожбанГрузинскийКарачаевскийКазахскийКхмерскийКорейскийКумыкскийКурдскийЛатинскийЛингалаЛитовскийЛатышскийМокшанскийМаориМарийскийМакедонскийМонгольскийМалайскийМальтийскийМайяЭрзянскийНидерландскийНорвежскийОсетинскийПенджабскийПалиПольскийПапьяментоДревнерусский языкПуштуПортугальскийКечуаКвеньяРумынский, МолдавскийРусскийЯкутскийСловацкийСловенскийАлбанскийСербскийШведскийСуахилиТамильскийТаджикскийТайскийТуркменскийТагальскийТурецкийТатарскийУдмурдскийУйгурскийУкраинскийУрдуУрумскийУзбекскийВодскийВьетнамскийВепсскийИдишЙорубаКитайский

решаем слау легко. Методы исключения гаусса

Пусть дана система , ∆≠0. (1)
Метод Гаусса – это метод последовательного исключения неизвестных.

Суть метода Гаусса состоит в преобразовании (1) к системе с треугольной матрицей , из которой затем последовательно (обратным ходом) получаются значения всех неизвестных. Рассмотрим одну из вычислительных схем. Эта схема называется схемой единственного деления. Итак, рассмотрим эту схему. Пусть a 11 ≠0 (ведущий элемент) разделим на a 11 первое уравнение. Получим
(2)
Пользуясь уравнением (2), легко исключить неизвестные x 1 из остальных уравнений системы (для этого достаточно из каждого уравнения вычесть уравнение (2) предварительно умноженное на соответствующий коэффициент при x 1), то есть на первом шаге получим
.
Иными словами, на 1 шаге каждый элемент последующих строк, начиная со второй, равен разности между исходным элементом и произведением его «проекции» на первый столбец и первую (преобразованную) строку.
Вслед за этим оставив первое уравнение в покое, над остальными уравнениями системы, полученной на первом шаге, совершим аналогичное преобразование: выберем из их числа уравнение с ведущим элементом и исключим с его помощью из остальных уравнений x 2 (шаг 2).
После n шагов вместо (1) получим равносильную систему
(3)
Таким образом, на первом этапе мы получим треугольную систему (3). Этот этап называется прямым ходом.
На втором этапе (обратный ход) мы находим последовательно из (3) значения x n , x n -1 , …, x 1 .
Обозначим полученное решение за x 0 . Тогда разность ε=b-A·x 0 называется невязкой .
Если ε=0, то найденное решение x 0 является верным.

Вычисления по методу Гаусса выполняются в два этапа:

  1. Первый этап называется прямым ходом метода. На первом этапе исходную систему преобразуют к треугольному виду.
  2. Второй этап называется обратным ходом. На втором этапе решают треугольную систему, эквивалентную исходной.
Коэффициенты а 11 , а 22 , …, называют ведущими элементами.
На каждом шаге предполагалось, что ведущий элемент отличен от нуля. Если это не так, то в качестве ведущего можно использовать любой другой элемент, как бы переставив уравнения системы.

Назначение метода Гаусса

Метод Гаусса предназначен для решения систем линейных уравнений. Относится к прямым методам решения.

Виды метода Гаусса

  1. Классический метод Гаусса;
  2. Модификации метода Гаусса. Одной из модификаций метода Гаусса является схема с выбором главного элемента. Особенностью метода Гаусса с выбором главного элемента является такая перестановка уравнений, чтобы на k -ом шаге ведущим элементом оказывался наибольший по модулю элемент k -го столбца.
  3. Метод Жордано-Гаусса;
Отличие метода Жордано-Гаусса от классического метода Гаусса состоит в применении правила прямоугольника , когда направление поиска решения происходит по главной диагонали (преобразование к единичной матрице). В методе Гаусса направление поиска решения происходит по столбцам (преобразование к системе с треугольной матрицей).
Проиллюстрируем отличие метода Жордано-Гаусса от метода Гаусса на примерах.

Пример решения методом Гаусса
Решим систему:

Для удобства вычислений поменяем строки местами:

Умножим 2-ую строку на (2). Добавим 3-ую строку к 2-ой

Умножим 2-ую строку на (-1). Добавим 2-ую строку к 1-ой

Из 1-ой строки выражаем x 3:
Из 2-ой строки выражаем x 2:
Из 3-ой строки выражаем x 1:

Пример решения методом Жордано-Гаусса
Эту же СЛАУ решим методом Жордано-Гаусса.

Последовательно будем выбирать разрешающий элемент РЭ, который лежит на главной диагонали матрицы.
Разрешающий элемент равен (1).

НЭ = СЭ – (А*В)/РЭ
РЭ – разрешающий элемент (1), А и В – элементы матрицы, образующие прямоугольник с элементами СТЭ и РЭ.
Представим расчет каждого элемента в виде таблицы:

x 1x 2x 3B
1 / 1 = 12 / 1 = 2-2 / 1 = -21 / 1 = 1


Разрешающий элемент равен (3).
На месте разрешающего элемента получаем 1, а в самом столбце записываем нули.
Все остальные элементы матрицы, включая элементы столбца B, определяются по правилу прямоугольника.
Для этого выбираем четыре числа, которые расположены в вершинах прямоугольника и всегда включают разрешающий элемент РЭ.
x 1x 2x 3B
0 / 3 = 03 / 3 = 11 / 3 = 0.334 / 3 = 1.33


Разрешающий элемент равен (-4).
На месте разрешающего элемента получаем 1, а в самом столбце записываем нули.
Все остальные элементы матрицы, включая элементы столбца B, определяются по правилу прямоугольника.
Для этого выбираем четыре числа, которые расположены в вершинах прямоугольника и всегда включают разрешающий элемент РЭ.
Представим расчет каждого элемента в виде таблицы:

Ответ : x 1 = 1, x 2 = 1, x 3 = 1

Реализация метода Гаусса

Метод Гаусса реализован на многих языках программирования, в частности: Pascal, C++, php, Delphi , а также имеется реализация метода Гаусса в онлайн режиме .

Использование метода Гаусса

Применение метода Гаусса в теории игр

В теории игр при отыскании максиминной оптимальной стратегии игрока составляется система уравнений, которая решается методом Гаусса.

Применение метода Гаусса при решении дифференциальных уравнений

Для поиска частного решения дифференциального уравнения сначала находят производные соответствующей степени для записанного частного решения (y=f(A,B,C,D)), которые подставляют в исходное уравнение. Далее, чтобы найти переменные A,B,C,D составляется система уравнений, которая решается методом Гаусса.

Применение метода Жордано-Гаусса в линейном программировании

В линейном программировании, в частности в симплекс-методе для преобразования симплексной таблицы на каждой итерации используется правило прямоугольника, в котором используется метод Жордано-Гаусса.

Рассмотрим один из самых распространенных методов решения систем линейных алгебраических уравнений – метод Гаусса. Этот метод (который называют также методом последовательного исключения неизвестных) известен в различных вариантах уже более 2000 лет.

Вычисления с помощью метода Гаусса состоят из двух основных этапов, называемых прямым ходом и обратным ходом (обратной подстановкой). Прямой ход метода Гаусса заключается в последовательном исключении неизвестных из системы (5.1) для преобразования ее к эквивалентной системе с верхней треугольной матрицей. Вычисления значений неизвестных производят на этапе обратного хода.

1. Схема единственного деления.

Рассмотрим сначала простейший вариант метода Гаусса, называемый схемой единственного деления.

Прямой ход состоит из шагов исключения.

1-й шаг. Целью этого шага является исключение неизвестного из уравнений с номерами Предположим, что коэффициент Будем называть его главным (или ведущим) элементом 1-го шага.

Найдем величины

называемые множителями 1-ю шага. Вычтем последовательно из второго, третьего, уравнений системы (5.1) первое уравнение, умноженное соответственно на Это позволит обратить в

нуль коэффициенты при во всех уравнениях, кроме первого. В результате получим эквивалентную систему

в которой вычисляются по формулам

2-й шаг. Целью этого шага является исключение неизвестного из уравнений с номерами Пусть где – коэффициент, называемый главным (или ведущим) элементом шага. Вычислим множители 2-го шага

и вычтем последовательно из третьего, четвертого, уравнений системы (5.30) второе уравнение, умноженное соответственно на . В результате получим систему

Здесь коэффициенты вычисляются по формулам

Аналогично проводятся остальные шаги. Опишем очередной шаг.

k-й шаг. В предположении, что главный (ведущий) элемент шага отличен от нуля, вычислим множители шага

и вычтем последовательно из уравнений полученной на предыдущем шаге системы уравнение, умноженное соответственно на

После шага исключения получим систему уравнений

матрица которой является верхней треугольной. На этом вычисления прямого хода заканчиваются.

Обратный ход. Из последнего уравнения системы (5.33) находим Подставляя найденное значение в предпоследнее уравнение, получим Осуществляя обратную подстановку, далее последовательно находим Вычисления неизвестных здесь проводятся по формулам

Трудоемкость метода. Оценим число арифметических операций, необходимых для реализации схемы единственного деления.

Вычисления 1-го шага исключения по формулам (5.29), (5.31) требуют выполнения деления, умножений и вычитаний, т. е. общее число арифметических операций составляет Аналогично, на шаге требуется операций, а на шаге – операций.

Подсчитаем теперь приближенно общее число арифметических операций прямого хода, считая размерность системы достаточно большой:

Как нетрудно видеть, для реализации обратного хода по формулам (5.34) нужно всего операций, что при больших пренебрежимо мало по сравнению с числом операций прямого хода.

Таким образом, для реализации метода Гаусса требуется примерно арифметических операций, причем подавляющее число этих действий совершается на этапе прямого хода.

Пример 5.7. Методом Гаусса решим систему

Прямой ход. 1-й шаг. Вычислим множители Вычитая из второго, третьего и четвертого уравнений системы (5.35) первое уравнение, умноженное на соответственно получим

2-й шаг. Вычислим множители Вычитая из третьего и четвертого уравнений системы (5.36) второе уравнение, умноженное на соответственно, приходим к системе

3-й шаг. Вычисляя множитель и вычитая из четвертого уравнения системы (5.37) третье уравнение, умноженное на приводим систему к треугольному виду:

Обратный ход. Из последнего уравнения системы находим Подставляя значение в третье уравнение, находим

Результаты вычислений можно свести в следующую таблицу.

Таблица 5.2 (см. скан)

Необходимость выбора главных элементов. Заметим, что вычисление множителей, а также обратная подстановка требуют деления на главные элементы Поэтому если один из главных элементов сказывается равным нулю, то схема единственного деления не может быть реализована. Здравый смысл подсказывает, что и в ситуации, когда все главные элементы отличны от нуля, но среди них есть близкие к нулю, возможен неконтролируемый рост погрешности.

Пример 5.8. Используя метод Гаусса, решим систему уравнений

на -разрядной десятичной ЭВМ.

Прямой ход. 1-й шаг. Вычисляем множители и преобразуем систему к виду

Все вычисления на этом шаге выполняются без округлений.

2-й шаг. После вычисления множителя последнее уравнение системы должно быть преобразовано к виду где Однако на используемой ЭВМ будет получено уравнение

Действительно, коэффициент определяется точно, так как при его вычислении не возникает чисел, мантиссы которых имеют более 6 разрядов. В то же время при вычислении умножение коэффициента 3.0001 на дает 7-разрядное число 105003.5, после округления которого до 6 разрядов получится 105004. Вычисление 62) завершается выполнением операции вычитания: . После округления последнего числа до 6 разрядов мантиссы приходим к уравнению (5.41).

Обратный ход. Из уравнения (5.41) находим и 1.00001. Сравнение с истинным значением показывает, что эта величина получена с очень высокой для используемой ЭВМ точностью. Дальнейшие вычисления дают

После округления имеем .

Как нетрудно видеть, найденные значения неизвестных имеют мало общего с истинными значениями решения

В чем же причина появления такой значительной погрешности? Говорить о накоплении ошибок округления не приходится, так как всего было выполнено 28 арифметических операций и лишь в 4 случаях потребовалось округление. Предположение о плохой обусловленности системы не подтверждается; вычисление дает значение и 100.

В действительности причина состоит в использовании на шаге малого ведущего элемента Следствием этого стало появление большого

множителя и существенное возрастание коэффициента в последнем уравнении системы.

Таким образом, изложенный выше вариант метода Гаусса (схема единственного деления) оказался некорректным и, следовательно, непригодным для вычислений на ЭВМ. Этот метод может привести к аварийному останову (если при некотором и вычисления по нему могут оказаться неустойчивыми.

2. Метод Гаусса с выбором главного элемента по столбцу (схема частичного выбора).

Описание метода. На шаге прямого хода коэффициенты уравнений системы с номерами преобразуются по формулам

Интуитивно ясно, что во избежание сильного роста коэффициентов системы и связанных с этим ошибок нельзя допускать появления больших множителей

В методе Гаусса с выбором главного элемента по столбцу гарантируется, что для всех к Отличие этого варианта метода Гаусса от схемы единственного деления заключается в том, что на шаге исключения в качестве главного элемента выбирают максимальный по модулю коэффициент а. при неизвестной в уравнениях с номерами Затем соответствующее выбранному коэффициенту уравнение с номером меняют местами с уравнением системы для того, чтобы главный элемент занял место коэффициента

После этой перестановки исключение неизвестного производят, как в схеме единственного деления.

Пример 5.9. Решим систему уравнений (5.39) методом Гаусса с выбором главного элемента по столбцу на -разрядной десятичной ЭВМ.

Прямой ход. 1-й шаг. Максимальный в первом столбце элемент матрицы находится в первой строке, поэтому перестановка уравнений не нужна. Здесь 1-й шаг проводится точно так же, как и в примере 5.8.

2-й шаг. Среди элементов матрицы системы (5.40) максимальный принадлежит третьему уравнению. Меняя местами второе и третье уравнения, получим систему

После вычисления последнее уравнение системы преобразуется к виду

Обратный ход. Из последнего уравнения находим Далее, имеем В данном случае ответ получился точным.

Заметим, что дополнительная работа по выбору главных элементов в схеме частичного выбора требует порядка действий, что практически не влияет на общую трудоемкость метода.

Вычислительная устойчивость схемы частичного выбора. Детальное исследование метода Гаусса показывает, что действительной причиной неустойчивости схемы единственного деления является возможность неограниченного роста элементов промежуточных матриц в процессе прямого хода. Так как на шаге схемы частичного выбора 1, то для вычисленных по формулам (5.42) элементов справедлива оценка Следовательно, максимальное по модулю значение элементов матрицы возрастает на одном шаге не более чем в 2 раза и в самом неблагоприятном случае шаг прямого хода даст коэффициент роста

Гарантия ограниченности роста элементов матрицы делает схему частичного выбора вычислительно устойчивой. Более того, для нее оказывается справедливой следующая оценка погрешности:

Здесь вычисленное на ЭВМ решение системы; его относительная погрешность; число обусловленности матрицы ем – машинное эпсилон; наконец, причем некоторая медленно растущая функция, зависящая от порядка системы (типа степенной функции с небольшим показателем), коэффициент роста.

Наличие в оценке (5.43) множителя указывает на то, что при большом схема частичного выбора может оказаться плохо обусловленной и возможна существенная потеря точности. Однако практика матричных вычислений показывает, что существенный рост элементов матрицы происходит крайне редко. В подавляющем большинстве случаев действительное значение коэффициента роста не превышает 8-10. Если система хорошо обусловлена, то погрешность вычисленного решения оказывается, как правило, малой.

Иногда для проверки качества приближенного решения х

вычисляют невязку и о степени близости приближенного решения к точному пытаются судить по тому, насколько мала невязка. Этот метод ненадежен по отношению к схеме частичного выбора, так как известно, что она гарантированно дает малые невдэки. Более точно это утверждение можно сформулировать так: справедлива оценка

где то же, что и в оценке (5.43). Заметим, что в неравенство (5.44) не входит число обусловленности.

3. Метод Гаусса с выборок главного элемента по всей матрице (схема полного выбора).

В этой схеме допускается нарушение естественного порядка исключения неизвестных.

На 1-м шаге метода среди элементов определяют максимальный по модулю элемент Первое уравнение системы и уравнение с номером меняют местами. Далее стандартным образом производят исключение неизвестного х, из всех уравнений, кроме первого. (что значительно меньше соответствующего значения для схемы частичного выбора). Подчеркнем, что до сих пор еще не найдено матрицы, для которой полный выбор дал бы значение Таким образом, для хорошо обусловленных систем этот вариант метода Гаусса является хорошо обусловленным.

Однако гарантия хорошей обусловленности достигается здесь ценой значительных затрат на выбор главных элементов. Для этого дополнительно к арифметических действий требуется произвести примерно операций сравнения, что может ощутимо замедлить процесс решения задачи на ЭВМ. Поэтому в большинстве случаев на практике предпочтение отдается все же схеме частичного выбора. Как уже отмечено, ситуации, когда при использовании этого варианта метода Гаусса происходит существенный рост элементов, встречаются чрезвычайно редко. Более того, эти ситуации могут быть легко выявлены с помощью заложенных в современных программах эффективных методов слежения за ростом элементов матриц.

4. Случаи, когда выбор главных элементов не нужен.

Известно, что для некоторых классов матриц при использовании схемы единственного деления главные элементы гарантированно располагаются на главной диагонали и потому применять частичный выбор нет необходимости. Так, например, обстоит дело для систем с положительно определенными матрицами, а также с матрицами, обладающими следующим свойством диагонального преобладания:

Матрицы, удовлетворяющие условию (5.45), таковы, что в каждой из строк модуль элемента расположенного на главной диагонали, больше суммы модулей всех остальных элементов строки.

5. Масштабирование.

Перед началом решения целесообразно масштабировать систему так, чтобы ее коэффициенты были величинами порядка единицы.

Существуют два естественных способа масштабирования системы Первый заключается в умножении каждого из уравнений на некоторый масштабирующий множитель Второй состоит в умножении на масштабирующий множитель каждого столбца матрицы, что соответствует замене переменных (фактически – это замена единиц измерения). В реальных ситуациях чаще всего масштабирование может быть выполнено без существенных трудностей. Однако подчеркнем, что в общем случае удовлетворительного способа масштабирования пока не найдено.

На практике масштабирование обычно производят с помощью деления каждого уравнения на его наибольший по модулю коэффициент. Это вполне удовлетворительный способ для большинства реально встречающихся задач.

Продолжаем рассматривать системы линейных уравнений. Этот урок является третьим по теме. Если вы смутно представляете, что такое система линейных уравнений вообще, чувствуете себя чайником, то рекомендую начать с азов на странице Далее полезно изучить урок .

Метод Гаусса – это просто! Почему? Известный немецкий математик Иоганн Карл Фридрих Гаусс еще при жизни получил признание величайшего математика всех времен, гения и даже прозвище «короля математики». А всё гениальное, как известно – просто! Кстати, на деньги попадают не только лохи, но еще и гении – портрет Гаусса красовался на купюре в 10 дойчмарок (до введения евро), и до сих пор Гаусс загадочно улыбается немцам с обычных почтовых марок.

Метод Гаусса прост тем, что для его освоения ДОСТАТОЧНО ЗНАНИЙ ПЯТИКЛАССНИКА.Необходимо уметь складывать и умножать! Не случайно метод последовательного исключения неизвестных преподаватели часто рассматривают на школьных математических факультативах. Парадокс, но у студентов метод Гаусса вызывает наибольшие сложности. Ничего удивительного – всё дело в методике, и я постараюсь в доступной форме рассказать об алгоритме метода.

Сначала немного систематизируем знания о системах линейных уравнений. Система линейных уравнений может:

1) Иметь единственное решение. 2) Иметь бесконечно много решений. 3) Не иметь решений (быть несовместной ).

Метод Гаусса – наиболее мощный и универсальный инструмент для нахождения решениялюбой системы линейных уравнений. Как мы помним, правило Крамера и матричный метод непригодны в тех случаях, когда система имеет бесконечно много решений или несовместна. А метод последовательного исключения неизвестных в любом случае приведет нас к ответу! На данном уроке мы опять рассмотрим метод Гаусса для случая №1 (единственное решение системы), под ситуации пунктов №№2-3 отведена статья. Замечу, что сам алгоритм метода во всех трёх случаях работает одинаково.

Вернемся к простейшей системе с урока Как решить систему линейных уравнений? и решим ее методом Гаусса.

На первом этапе нужно записать расширенную матрицу системы : . По какому принципу записаны коэффициенты, думаю, всем видно. Вертикальная черта внутри матрицы не несёт никакого математического смысла – это просто отчеркивание для удобства оформления.

Справка : рекомендую запомнить термины линейной алгебры. Матрица системы – это матрица, составленная только из коэффициентов при неизвестных, в данном примере матрица системы: . Расширенная матрица системы – это та же матрица системы плюс столбец свободных членов, в данном случае: . Любую из матриц можно для краткости называть просто матрицей.

После того, как расширенная матрица системы записана, с ней необходимо выполнить некоторые действия, которые также называются элементарными преобразованиями .

Существуют следующие элементарные преобразования:

1) Строки матрицы можно переставлять местами. Например, в рассматриваемой матрице можно безболезненно переставить первую и вторую строки:

2) Если в матрице есть (или появились) пропорциональные (как частный случай – одинаковые) строки, то следует удалить из матрицы все эти строки кроме одной. Рассмотрим, например матрицу . В данной матрице последние три строки пропорциональны, поэтому достаточно оставить только одну из них: .

3) Если в матрице в ходе преобразований появилась нулевая строка, то ее также следуетудалить . Рисовать не буду, понятно, нулевая строка – это строка, в которой одни нули .

4) Строку матрицы можно умножить (разделить) на любое число, отличное от нуля . Рассмотрим, например, матрицу . Здесь целесообразно первую строку разделить на –3, а вторую строку – умножить на 2: . Данное действие очень полезно, поскольку упрощает дальнейшие преобразования матрицы.

5) Это преобразование вызывает наибольшие затруднения, но на самом деле ничего сложного тоже нет. К строке матрицы можно прибавить другую строку, умноженную на число , отличное от нуля. Рассмотрим нашу матрицу из практического примера: . Сначала я распишу преобразование очень подробно. Умножаем первую строку на –2: , и ко второй строке прибавляем первую строку умноженную на –2 : . Теперь первую строку можно разделить «обратно» на –2: . Как видите, строка, которую ПРИБАВЛЯЛИ не изменилась . Всегда меняется строка, К КОТОРОЙ ПРИБАВЛЯЮТ .

На практике так подробно, конечно, не расписывают, а пишут короче: Еще раз: ко второй строке прибавили первую строку, умноженную на –2 . Умножают строку обычно устно или на черновике, при этом мысленный ход расчётов примерно такой:

«Переписываю матрицу и переписываю первую строку: »

«Сначала первый столбец. Внизу мне нужно получить ноль. Поэтому единицу вверху умножаю на –2: , и ко второй строке прибавляю первую: 2 + (–2) = 0. Записываю результат во вторую строку: »

«Теперь второй столбец. Вверху –1 умножаю на –2: . Ко второй строке прибавляю первую: 1 + 2 = 3. Записываю результат во вторую строку: »

«И третий столбец. Вверху –5 умножаю на –2: . Ко второй строке прибавляю первую: –7 + 10 = 3. Записываю результат во вторую строку: »

Пожалуйста, тщательно осмыслите этот пример и разберитесь в последовательном алгоритме вычислений, если вы это поняли, то метод Гаусса практически «в кармане». Но, конечно, над этим преобразованием мы еще поработаем.

Элементарные преобразования не меняют решение системы уравнений

! ВНИМАНИЕ : рассмотренные манипуляции нельзя использовать , если Вам предложено задание, где матрицы даны «сами по себе». Например, при «классических» действиях с матрицами что-то переставлять внутри матриц ни в коем случае нельзя! Вернемся к нашей системе . Она практически разобрана по косточкам.

Запишем расширенную матрицу системы и с помощью элементарных преобразований приведем ее к ступенчатому виду :

(1) Ко второй строке прибавили первую строку, умноженную на –2. И снова: почему первую строку умножаем именно на –2? Для того чтобы внизу получить ноль, а значит, избавиться от одной переменной во второй строке.

(2) Делим вторую строку на 3.

Цель элементарных преобразований привести матрицу к ступенчатому виду: . В оформлении задания прямо так и отчеркивают простым карандашом «лестницу», а также обводят кружочками числа, которые располагаются на «ступеньках». Сам термин «ступенчатый вид» не вполне теоретический, в научной и учебной литературе он часто называется трапециевидный вид или треугольный вид .

В результате элементарных преобразований получена эквивалентная исходной система уравнений:

Теперь систему нужно «раскрутить» в обратном направлении – снизу вверх, этот процесс называется обратным ходом метода Гаусса .

В нижнем уравнении у нас уже готовый результат: .

Рассмотрим первое уравнение системы и подставим в него уже известное значение «игрек»:

Рассмотрим наиболее распространенную ситуацию, когда методом Гаусса требуется решить систему трёх линейных уравнений с тремя неизвестными.

Пример 1

Решить методом Гаусса систему уравнений:

Запишем расширенную матрицу системы:

Сейчас я сразу нарисую результат, к которому мы придём в ходе решения: И повторюсь, наша цель – с помощью элементарных преобразований привести матрицу к ступенчатому виду. С чего начать действия?

Сначала смотрим на левое верхнее число: Почти всегда здесь должна находиться единица . Вообще говоря, устроит и –1 (а иногда и другие числа), но как-то так традиционно сложилось, что туда обычно помещают единицу. Как организовать единицу? Смотрим на первый столбец – готовая единица у нас есть! Преобразование первое: меняем местами первую и третью строки:

Теперь первая строка у нас останется неизменной до конца решения . Уже легче.

Единица в левом верхнем углу организована. Теперь нужно получить нули вот на этих местах:

Нули получаем как раз с помощью «трудного» преобразования. Сначала разбираемся со второй строкой (2, –1, 3, 13). Что нужно сделать, чтобы на первой позиции получить ноль? Нужно ко второй строке прибавить первую строку, умноженную на –2 . Мысленно или на черновике умножаем первую строку на –2: (–2, –4, 2, –18). И последовательно проводим (опять же мысленно или на черновике) сложение, ко второй строке прибавляем первую строку, уже умноженную на –2 :

Результат записываем во вторую строку:

Аналогично разбираемся с третьей строкой (3, 2, –5, –1). Чтобы получить на первой позиции ноль, нужно к третьей строке прибавить первую строку, умноженную на –3 . Мысленно или на черновике умножаем первую строку на –3: (–3, –6, 3, –27). И к третьей строке прибавляем первую строку, умноженную на –3 :

Результат записываем в третью строку:

На практике эти действия обычно выполняются устно и записываются в один шаг:

Не нужно считать всё сразу и одновременно . Порядок вычислений и «вписывания» результатов последователен и обычно такой: сначала переписываем первую строку, и пыхтим себе потихонечку – ПОСЛЕДОВАТЕЛЬНО иВНИМАТЕЛЬНО :
А мысленный ход самих расчётов я уже рассмотрел выше.

В данном примере это сделать легко, вторую строку делим на –5 (поскольку там все числа делятся на 5 без остатка). Заодно делим третью строку на –2, ведь чем меньше числа, тем проще решение:

На заключительном этапе элементарных преобразований нужно получить еще один ноль здесь:

Для этого к третьей строке прибавляем вторую строку, умноженную на –2 :
Попробуйте разобрать это действие самостоятельно – мысленно умножьте вторую строку на –2 и проведите сложение.

Последнее выполненное действие – причёска результата, делим третью строку на 3.

В результате элементарных преобразований получена эквивалентная исходной система линейных уравнений: Круто.

Теперь в действие вступает обратный ход метода Гаусса. Уравнения «раскручиваются» снизу вверх.

В третьем уравнении у нас уже готовый результат:

Смотрим на второе уравнение: . Значение «зет» уже известно, таким образом:

И, наконец, первое уравнение: . «Игрек» и «зет» известны, дело за малым:

Ответ :

Как уже неоднократно отмечалось, для любой системы уравнений можно и нужно сделать проверку найденного решения, благо, это несложно и быстро.

Пример 2

Это пример для самостоятельного решения, образец чистового оформления и ответ в конце урока.

Следует отметить, что ваш ход решения может не совпасть с моим ходом решения, и это – особенность метода Гаусса . Но вот ответы обязательно должны получиться одинаковыми!

Пример 3

Решить систему линейных уравнений методом Гаусса

Смотрим на левую верхнюю «ступеньку». Там у нас должна быть единица. Проблема состоит в том, что в первом столбце единиц нет вообще, поэтому перестановкой строк ничего не решить. В таких случаях единицу нужно организовать с помощью элементарного преобразования. Обычно это можно сделать несколькими способами. Я поступил так: (1) К первой строке прибавляем вторую строку, умноженную на –1 . То есть, мысленно умножили вторую строку на –1 и выполнили сложение первой и второй строки, при этом вторая строка у нас не изменилась.

Теперь слева вверху «минус один», что нас вполне устроит. Кто хочет получить +1, может выполнить дополнительное телодвижение: умножить первую строку на –1 (сменить у неё знак).

(2) Ко второй строке прибавили первую строку, умноженную на 5. К третьей строке прибавили первую строку, умноженную на 3.

(3) Первую строку умножили на –1, в принципе, это для красоты. У третьей строки также сменили знак и переставили её на второе место, таким образом, на второй «ступеньке у нас появилась нужная единица.

(4) К третьей строке прибавили вторую строку, умноженную на 2.

(5) Третью строку разделили на 3.

Скверным признаком, который свидетельствует об ошибке в вычислениях (реже – об опечатке), является «плохая» нижняя строка. То есть, если бы у нас внизу получилось что-нибудь вроде , и, соответственно, , то с большой долей вероятности можно утверждать, что допущена ошибка в ходе элементарных преобразований.

Заряжаем обратный ход, в оформлении примеров часто не переписывают саму систему, а уравнения «берут прямо из приведенной матрицы». Обратный ход, напоминаю, работает, снизу вверх. Да тут подарок получился:

Ответ : .

Пример 4

Решить систему линейных уравнений методом Гаусса

Это пример для самостоятельного решения, он несколько сложнее. Ничего страшного, если кто-нибудь запутается. Полное решение и образец оформления в конце урока. Ваше решение может отличаться от моего решения.

В последней части рассмотрим некоторые особенности алгоритма Гаусса. Первая особенность состоит в том, что иногда в уравнениях системы отсутствуют некоторые переменные, например: Как правильно записать расширенную матрицу системы? Об этом моменте я уже рассказывал на уроке Правило Крамера. Матричный метод . В расширенной матрице системы на месте отсутствующих переменных ставим нули: Кстати, это довольно легкий пример, поскольку в первом столбце уже есть один ноль, и предстоит выполнить меньше элементарных преобразований.

Вторая особенность состоит вот в чём. Во всех рассмотренных примерах на «ступеньки» мы помещали либо –1, либо +1. Могут ли там быть другие числа? В ряде случаев могут. Рассмотрим систему: .

Здесь на левой верхней «ступеньке» у нас двойка. Но замечаем тот факт, что все числа в первом столбце делятся на 2 без остатка – и другая двойка и шестерка. И двойка слева вверху нас устроит! На первом шаге нужно выполнить следующие преобразования: ко второй строке прибавить первую строку, умноженную на –1; к третьей строке прибавить первую строку, умноженную на –3. Таким образом, мы получим нужные нули в первом столбце.

Или еще такой условный пример: . Здесь тройка на второй «ступеньке» тоже нас устраивает, поскольку 12 (место, где нам нужно получить ноль) делится на 3 без остатка. Необходимо провести следующее преобразование: к третьей строке прибавить вторую строку, умноженную на –4, в результате чего и будет получен нужный нам ноль.

Метод Гаусса универсален, но есть одно своеобразие. Уверенно научиться решать системы другими методами (методом Крамера, матричным методом) можно буквально с первого раза – там очень жесткий алгоритм. Но вот чтобы уверенно себя чувствовать в методе Гаусса, следует «набить руку», и прорешать хотя бы 5-10 десять систем. Поэтому поначалу возможны путаница, ошибки в вычислениях, и в этом нет ничего необычного или трагического.

Дождливая осенняя погода за окном…. Поэтому для всех желающих более сложный пример для самостоятельного решения:

Пример 5

Решить методом Гаусса систему 4-х линейных уравнений с четырьмя неизвестными.

Такое задание на практике встречается не так уж и редко. Думаю, даже чайнику, который обстоятельно изучил эту страницу, интуитивно понятен алгоритм решения такой системы. Принципиально всё так же – просто действий больше.

Случаи, когда система не имеет решений (несовместна) или имеет бесконечно много решений, рассмотрены на уроке Несовместные системы и системы с общим решением . Там же можно закрепить рассмотренный алгоритм метода Гаусса.

Желаю успехов!

Решения и ответы:

Пример 2: Решение : Запишем расширенную матрицу системы и с помощью элементарных преобразований приведем ее к ступенчатому виду.
Выполненные элементарные преобразования: (1) Ко второй строке прибавили первую строку, умноженную на –2. К третьей строке прибавили первую строку, умноженную на –1. Внимание! Здесь может возникнуть соблазн из третьей строки вычесть первую, крайне не рекомендую вычитать – сильно повышается риск ошибки. Только складываем! (2) У второй строки сменили знак (умножили на –1). Вторую и третью строки поменяли местами. Обратите внимание , что на «ступеньках» нас устраивает не только единица, но еще и –1, что даже удобнее. (3) К третьей строке прибавили вторую строку, умноженную на 5. (4) У второй строки сменили знак (умножили на –1). Третью строку разделили на 14.

Обратный ход:

Ответ : .

Пример 4: Решение : Запишем расширенную матрицу системы и с помощью элементарных преобразований приведем ее к ступенчатому виду:

Выполненные преобразования: (1) К первой строке прибавили вторую. Таким образом, организована нужная единица на левой верхней «ступеньке». (2) Ко второй строке прибавили первую строку, умноженную на 7. К третьей строке прибавили первую строку, умноженную на 6.

Со второй «ступенькой» всё хуже , «кандидаты» на неё – числа 17 и 23, а нам нужна либо единичка, либо –1. Преобразования (3) и (4) будут направлены на получение нужной единицы (3) К третьей строке прибавили вторую, умноженную на –1. (4) Ко второй строке прибавили третью, умноженную на –3. Нужная вещь на второй ступеньке получена . (5) К третьей строке прибавили вторую, умноженную на 6. (6) Вторую строку умножили на –1, третью строку разделили на -83.

Обратный ход:

Ответ :

Пример 5: Решение : Запишем матрицу системы и с помощью элементарных преобразований приведем ее к ступенчатому виду:

Выполненные преобразования: (1) Первую и вторую строки поменяли местами. (2) Ко второй строке прибавили первую строку, умноженную на –2. К третьей строке прибавили первую строку, умноженную на –2. К четвертой строке прибавили первую строку, умноженную на –3. (3) К третьей строке прибавили вторую, умноженную на 4. К четвертой строке прибавили вторую, умноженную на –1. (4) У второй строки сменили знак. Четвертую строку разделили на 3 и поместили вместо третьей строки. (5) К четвертой строке прибавили третью строку, умноженную на –5.

Обратный ход:

Ответ :

Метод Гаусса – метод последовательного исключения неизвестных – заключается в том, что с помощью элементарных преобразований исходная система приводится к равносильной ей системе ступенчатого или треугольного вида, из которой последовательно, начиная с последних (по номеру), неизвестных находятся все остальные неизвестные. Дана система (1)

Начинаем осуществлять прямой ход . Считаем, что коэффициент а 11 ≠ 0; если же это не так, меняем местами уравнения.

Первый шаг состоит в том, чтобы исключить неизвестное х 1 из всех уравнений, кроме первого. Для этого ко второму уравнению прибавим первое уравнение, умноженное на число
, к третьему уравнению прибавим первое уравнение, умноженное на число
, и так далее до последнего уравнения. После первого шага получим систему:

Полученная система равносильна исходной системе.

Вторым шагом исключают неизвестное из всех уравнений, кроме первого и второго. Для этого повторяем все действия первого шага для второго и последующих уравнений, а именно: считаем, что коэффициент
≠ 0 и так далее. Если в результате преобразований получается нулевое уравнение, то его удаляют, если же получается несовместное уравнение, то решение системы закончено – она несовместна. Процесс исключения неизвестных продолжаем до тех пор, пока это возможно. Обозначим количество уравнений, оставшихся после прямого хода, через r . Это число равно рангу основной матрицы системы и может быть меньше или равно n . Рассмотрим оба случая.

1) Если r = n

где с 11 ≠ 0, с 22 ≠ 0, …, с nn ≠ 0.

Обратным ходом , начиная с последнего уравнения, последовательно найдем значения x n , (где x n = ), x n – 1 , …, x 1 . В этом случае система линейных уравнений имеет единственное решение, то есть является определенной.

2) Если r n , то система после прямого хода принимает вид:

где с 11 ≠ 0, с 22 ≠ 0, …, с rr ≠ 0. Неизвестные x 1 , x 2 , …, x r , с которых начинаются уравнения, называются главными неизвестными , а остальные x r + 1 , x r + 2 , …, x n свободными . В этом случае обратным ходом, начиная с последнего уравнения, выражают главные неизвестные через свободные неизвестные. Получают следующие равенства:

x 1 = k 1, r + 1 x r + 1 + … + k 1, n x n + t 1 ,

x 2 = k 2, r + 1 x r + 1 + … + k 2, n x n + t 2 ,

……………………………………..

x r = k r , r + 1 x r + 1 + … + k r , n x n + t r .

Определение 6.10. Общим решением системы называется выражение главных неизвестных через свободные.

Если свободным неизвестным придать какие-нибудь числовые значения, то из общего решения получим значения главных неизвестных. Таким образом, получают частное решение системы. Из способа его получения следует, что система имеет более одного решения, то есть является неопределенной.

Пример 6.3. Решить методом Гаусса систему линейных уравнений:

Решение . Преобразования с системой линейных удобнее производить не с самими уравнениями, а с матрицей их коэффициентов. Расширенная матрица этой системы имеет вид: (А |B ) =
.

Осуществляем прямой ход. Первым шагом исключаем неизвестное х 1 из всех уравнений, кроме первого. Так как а 11 = 1 ≠ 0, то переставлять уравнения местами не нужно. Прибавим ко второму уравнению системы первое уравнение, умноженное на (–1), к третьему уравнению – первое, умноженное на (–3). Получим после преобразований следующую матрицу:
, в которой элемент а 22 = 1. Перестановка местами уравнений (первое уравнение трогать не следует) не поможет, поэтому переходим к следующему неизвестному х 3 и исключаем его из всех уравнений, кроме первого и второго. Для этого к третьему уравнению прибавим второе, умноженное на (–2) и вычеркнем получившееся нулевое уравнение. После прямого хода получаем следующую систему:
. Прямой ход завершен. В этом случае n = 4, r = 2, r n , и, следовательно, система неопределенная. Главные неизвестные – это те неизвестные, с которых начинаются уравнения, в нашем случае это х 1 и х 3 . Неизвестные х 2 и х 4 – свободные.

Обратным ходом надо выразить главные неизвестные через свободные. Для этого в столбцах, содержащих ведущие элементы строк, следует получить нули. Здесь это элемент а 13 . Прибавим к первому уравнению, умноженному на 2, второе и выпишем получившуюся матрицу коэффициентов:
, а затем и сами уравнения:
Из этих уравнений получаем общее решение:

Найдем какое-нибудь частное решение; пусть х 2 = 3, х 4 = 1, тогда из общего решения получим значения х 1 = , и х 1 = –2. Таким образом, частное решение – вектор а = (, 3, –2, 1).

Ответ : общее решение {(
, х 2 ,
, х 4)}, где х 2 , х 4  R;

частное решение, если х 2 = 3, х 4 = 1, то (, 3, –2, 1).

Метод Гаусса прекрасно подходит для решения систем линейных алгебраических уравнений (СЛАУ). Он обладает рядом преимуществ по сравнению с другими методами:

  • во-первых, нет необходимости предварительно исследовать систему уравнений на совместность;
  • во-вторых, методом Гаусса можно решать не только СЛАУ, в которых число уравнений совпадает с количеством неизвестных переменных и основная матрица системы невырожденная, но и системы уравнений, в которых число уравнений не совпадает с количеством неизвестных переменных или определитель основной матрицы равен нулю;
  • в-третьих, метод Гаусса приводит к результату при сравнительно небольшом количестве вычислительных операций.

Краткий обзор статьи.

Сначала дадим необходимые определения и введем обозначения.

Далее опишем алгоритм метода Гаусса для простейшего случая, то есть, для систем линейных алгебраических уравнений, количество уравнений в которых совпадает с количеством неизвестных переменных и определитель основной матрицы системы не равен нулю. При решении таких систем уравнений наиболее отчетливо видна суть метода Гаусса, которая заключается в последовательном исключении неизвестных переменных. Поэтому метод Гаусса также называют методом последовательного исключения неизвестных. Покажем подробные решения нескольких примеров.

В заключении рассмотрим решение методом Гаусса систем линейных алгебраических уравнений, основная матрица которых либо прямоугольная, либо вырожденная. Решение таких систем имеет некоторые особенности, которые мы подробно разберем на примерах.

Навигация по странице.

Основные определения и обозначения.

Рассмотрим систему из p линейных уравнений с n неизвестными (p может быть равно n ):

Где – неизвестные переменные, – числа (действительные или комплексные), – свободные члены.

Если , то система линейных алгебраических уравнений называется однородной , в противном случае – неоднородной .

Совокупность значения неизвестных переменных , при которых все уравнения системы обращаются в тождества, называется решением СЛАУ .

Если существует хотя бы одно решение системы линейных алгебраических уравнений, то она называется совместной , в противном случае – несовместной .

Если СЛАУ имеет единственное решение, то она называется определенной . Если решений больше одного, то система называется неопределенной .

Говорят, что система записана в координатной форме , если она имеет вид
.

Эта система в матричной форме записи имеет вид , где – основная матрица СЛАУ, – матрица столбец неизвестных переменных, – матрица свободных членов.

Если к матрице А добавить в качестве (n+1)-ого столбца матрицу-столбец свободных членов, то получим так называемую расширенную матрицу системы линейных уравнений. Обычно расширенную матрицу обозначают буквой Т , а столбец свободных членов отделяют вертикальной линией от остальных столбцов, то есть,

Квадратная матрица А называется вырожденной , если ее определитель равен нулю. Если , то матрица А называется невырожденной .

Следует оговорить следующий момент.

Если с системой линейных алгебраических уравнений произвести следующие действия

  • поменять местами два уравнения,
  • умножить обе части какого-либо уравнения на произвольное и отличное от нуля действительное (или комплексное) число k ,
  • к обеим частям какого-либо уравнения прибавить соответствующие части другого уравнения, умноженные на произвольное число k ,

то получится эквивалентная система, которая имеет такие же решения (или также как и исходная не имеет решений).

Для расширенной матрицы системы линейных алгебраических уравнений эти действия будут означать проведение элементарных преобразований со строками:

  • перестановку двух строк местами,
  • умножение всех элементов какой-либо строки матрицы T на отличное от нуля число k ,
  • прибавление к элементам какой-либо строки матрицы соответствующих элементов другой строки, умноженных на произвольное число k .

Теперь можно переходить к описанию метода Гаусса.

Решение систем линейных алгебраических уравнений, в которых число уравнений равно числу неизвестных и основная матрица системы невырожденная, методом Гаусса.

Как бы мы поступили в школе, если бы получили задание найти решение системы уравнений .

Некоторые сделали бы так.

Заметим, что прибавив к левой части второго уравнения левую часть первого, а к правой части – правую, можно избавиться от неизвестных переменных x 2 и x 3 и сразу найти x 1 :

Подставляем найденное значение x 1 =1 в первое и третье уравнение системы:

Если умножить обе части третьего уравнения системы на -1 и прибавить их к соответствующим частям первого уравнения, то мы избавимся от неизвестной переменной x 3 и сможем найти x 2 :

Подставляем полученное значение x 2 =2 в третье уравнение и находим оставшуюся неизвестную переменную x 3 :

Другие поступили бы иначе.

Разрешим первое уравнение системы относительно неизвестной переменной x 1 и подставим полученное выражение во второе и третье уравнение системы, чтобы исключить из них эту переменную:

Теперь разрешим второе уравнение системы относительно x 2 и подставим полученный результат в третье уравнение, чтобы исключить из него неизвестную переменную x 2 :

Из третьего уравнения системы видно, что x 3 =3 . Из второго уравнения находим , а из первого уравнения получаем .

Знакомые способы решения, не правда ли?

Самое интересное здесь то, что второй способ решения по сути и есть метод последовательного исключения неизвестных, то есть, метод Гаусса. Когда мы выражали неизвестные переменные (сначала x 1 , на следующем этапе x 2 ) и подставляли их в остальные уравнения системы, мы тем самым исключали их. Исключение мы проводили до того момента, пока в последнем уравнении не осталась одна единственная неизвестная переменная. Процесс последовательного исключения неизвестных называется прямым ходом метода Гаусса . После завершения прямого хода у нас появляется возможность вычислить неизвестную переменную, находящуюся в последнем уравнении. С ее помощью из предпоследнего уравнения находим следующую неизвестную переменную и так далее. Процесс последовательного нахождения неизвестных переменных при движении от последнего уравнения к первому называется обратным ходом метода Гаусса .

Следует заметить, что когда мы выражаем x 1 через x 2 и x 3 в первом уравнении, а затем подставляем полученное выражение во второе и третье уравнения, то к такому же результату приводят следующие действия:

Действительно, такая процедура также позволяет исключить неизвестную переменную x 1 из второго и третьего уравнений системы:

Нюансы с исключением неизвестных переменных по методу Гаусса возникают тогда, когда уравнения системы не содержат некоторых переменных.

Например, в СЛАУ в первом уравнении отсутствует неизвестная переменная x 1 (иными словами, коэффициент перед ней равен нулю). Поэтому мы не можем разрешить первое уравнение системы относительно x 1 , чтобы исключить эту неизвестную переменную из остальных уравнений. Выходом из этой ситуации является перестановка местами уравнений системы. Так как мы рассматриваем системы линейных уравнений, определители основных матриц которых отличны от нуля, то всегда существует уравнение, в котором присутствует нужная нам переменная, и мы это уравнение можем переставить на нужную нам позицию. Для нашего примера достаточно поменять местами первое и второе уравнения системы , дальше можно разрешить первое уравнение относительно x 1 и исключить ее из остальных уравнений системы (хотя во втором уравнении x 1 уже отсутствует).

Надеемся, что суть Вы уловили.

Опишем алгоритм метода Гаусса.

Пусть нам требуется решить систему из n линейных алгебраических уравнений с n неизвестными переменными вида , и пусть определитель ее основной матрицы отличен от нуля.

Будем считать, что , так как мы всегда можем этого добиться перестановкой местами уравнений системы. Исключим неизвестную переменную x 1 из всех уравнений системы, начиная со второго. Для этого ко второму уравнению системы прибавим первое, умноженное на , к третьему уравнению прибавим первое, умноженное на , и так далее, к n-ому уравнению прибавим первое, умноженное на . Система уравнений после таких преобразований примет вид

где , а .

К такому же результату мы бы пришли, если бы выразили x 1 через другие неизвестные переменные в первом уравнении системы и полученное выражение подставили во все остальные уравнения. Таким образом, переменная x 1 исключена из всех уравнений, начиная со второго.

Далее действуем аналогично, но лишь с частью полученной системы, которая отмечена на рисунке

Для этого к третьему уравнению системы прибавим второе, умноженное на , к четвертому уравнению прибавим второе, умноженное на , и так далее, к n-ому уравнению прибавим второе, умноженное на . Система уравнений после таких преобразований примет вид

где , а . Таким образом, переменная x 2 исключена из всех уравнений, начиная с третьего.

Далее приступаем к исключению неизвестной x 3 , при этом действуем аналогично с отмеченной на рисунке частью системы

Так продолжаем прямой ход метода Гаусса пока система не примет вид

С этого момента начинаем обратный ход метода Гаусса: вычисляем x n из последнего уравнения как , с помощью полученного значения x n находим x n-1 из предпоследнего уравнения, и так далее, находим x 1 из первого уравнения.

Разберем алгоритм на примере.

Пример.

методом Гаусса.

Решение.

Коэффициент a 11 отличен от нуля, так что приступим к прямому ходу метода Гаусса, то есть, к исключению неизвестной переменной x 1 из всех уравнений системы, кроме первого. Для этого к левой и правой частям второго, третьего и четвертого уравнения прибавим левую и правую части первого уравнения, умноженные соответственно на , и :

Неизвестную переменную x 1 исключили, переходим к исключению x 2 . К левым и правым частям третьего и четвертого уравнений системы прибавляем левую и правую части второго уравнения, умноженные соответственно на и :

Для завершения прямого хода метода Гаусса нам осталось исключить неизвестную переменную x 3 из последнего уравнения системы. Прибавим к левой и правой частям четвертого уравнения соответственно левую и правую часть третьего уравнения, умноженную на :

Можно начинать обратный ход метода Гаусса.

Из последнего уравнения имеем ,
из третьего уравнения получаем ,
из второго ,
из первого .

Для проверки можно подставить полученные значения неизвестных переменных в исходную систему уравнений. Все уравнения обращаются в тождества, что говорит о том, что решение по методу Гаусса найдено верно.

Ответ:

А сейчас приведем решение этого же примера методом Гаусса в матричной форме записи.

Пример.

Найдите решение системы уравнений методом Гаусса.

Решение.

Расширенная матрица системы имеет вид . Сверху над каждым столбцом записаны неизвестные переменные, которым соответствуют элементы матрицы.

Прямой ход метода Гаусса здесь предполагает приведение расширенной матрицы системы к трапецеидальному виду с помощью элементарных преобразований. Этот процесс схож с исключением неизвестных переменных, которое мы проводили с системой в координатной форме. Сейчас Вы в этом убедитесь.

Преобразуем матрицу так, чтобы все элементы в первом столбце, начиная со второго, стали нулевыми. Для этого к элементам второй, третьей и четвертой строк прибавим соответствующие элементы первой строки умноженные на , и на соответственно:

Далее полученную матрицу преобразуем так, чтобы во втором столбце все элементы, начиная с третьего стали нулевыми. Это будет соответствовать исключению неизвестной переменной x 2 . Для этого к элементам третьей и четвертой строк прибавим соответствующие элементы первой строки матрицы, умноженные соответственно на и :

Осталось исключить неизвестную переменную x 3 из последнего уравнения системы. Для этого к элементам последней строки полученной матрицы прибавим соответствующие элементы предпоследней строки, умноженные на :

Следует отметить, что эта матрица соответствует системе линейных уравнений

которая была получена ранее после прямого хода.

Пришло время обратного хода. В матричной форме записи обратный ход метода Гаусса предполагает такое преобразование полученной матрицы, чтобы матрица, отмеченная на рисунке

стала диагональной, то есть, приняла вид

где – некоторые числа.

Эти преобразования аналогичны преобразованиям прямого хода метода Гаусса, но выполняются не от первой строки к последней, а от последней к первой.

Прибавим к элементам третьей, второй и первой строк соответствующие элементы последней строки, умноженные на , на и на соответственно:

Теперь прибавим к элементам второй и первой строк соответствующие элементы третьей строки, умноженные на и на соответственно:

На последнем шаге обратного хода метода Гаусса к элементам первой строки прибавляем соответствующие элементы второй строки, умноженные на :

Полученная матрица соответствует системе уравнений , откуда находим неизвестные переменные.

Ответ:

ОБРАТИТЕ ВНИМАНИЕ.

При использовании метода Гаусса для решения систем линейных алгебраических уравнений следует избегать приближенных вычислений, так как это может привести к абсолютно неверным результатам. Рекомендуем не округлять десятичные дроби. Лучше от десятичных дробей переходить к обыкновенным дробям.

Пример.

Решите систему из трех уравнений методом Гаусса .

Решение.

Отметим, что в этом примере неизвестные переменные имеют другое обозначение (не x 1 , x 2 , x 3 , а x, y, z ). Перейдем к обыкновенным дробям:

Исключим неизвестную x из второго и третьего уравнений системы:

В полученной системе во втором уравнении отсутствует неизвестная переменная y , а в третьем уравнении y присутствует, поэтому, переставим местами второе и третье уравнения:

На этом прямой ход метода Гаусса закончен (из третьего уравнения не нужно исключать y , так как этой неизвестной переменной уже нет).

Приступаем к обратному ходу.

Из последнего уравнения находим ,
из предпоследнего


из первого уравнения имеем

Ответ:

X = 10, y = 5, z = -20 .

Решение систем линейных алгебраических уравнений, в которых число уравнений не совпадает с числом неизвестных или основная матрица системы вырожденная, методом Гаусса.

Системы уравнений, основная матрица которых прямоугольная или квадратная вырожденная, могут не иметь решений, могут иметь единственное решение, а могут иметь бесконечное множество решений.

Сейчас мы разберемся, как метод Гаусса позволяет установить совместность или несовместность системы линейных уравнений, а в случае ее совместности определить все решения (или одно единственное решение).

В принципе процесс исключения неизвестных переменных в случае таких СЛАУ остается таким же. Однако следует подробно остановиться на некоторых ситуациях, которые могут возникнуть.

Переходим к самому важному этапу.

Итак, допустим, что система линейных алгебраических уравнений после завершения прямого хода метода Гаусса приняла вид и ни одно уравнение не свелось к (в этом случае мы бы сделали вывод о несовместности системы). Возникает логичный вопрос: «Что делать дальше»?

Выпишем неизвестные переменные, которые стоят на первом месте всех уравнений полученной системы:

В нашем примере это x 1 , x 4 и x 5 . В левых частях уравнений системы оставляем только те слагаемые, которые содержат выписанные неизвестные переменные x 1 , x 4 и x 5 , остальные слагаемые переносим в правую часть уравнений с противоположным знаком:

Придадим неизвестным переменным, которые находятся в правых частях уравнений, произвольные значения , где – произвольные числа:

После этого в правых частях всех уравнений нашей СЛАУ находятся числа и можно преступать к обратному ходу метода Гаусса.

Из последнего уравнений системы имеем , из предпоследнего уравнения находим , из первого уравнения получаем

Решением системы уравнений является совокупность значений неизвестных переменных

Придавая числам различные значения, мы будем получать различные решения системы уравнений. То есть, наша система уравнений имеет бесконечно много решений.

Ответ:

где – произвольные числа.

Для закрепления материала подробно разберем решения еще нескольких примеров.

Пример.

Решите однородную систему линейных алгебраических уравнений методом Гаусса.

Решение.

Исключим неизвестную переменную x из второго и третьего уравнений системы. Для этого к левой и правой части второго уравнения прибавим соответственно левую и правую части первого уравнения, умноженные на , а к левой и правой части третьего уравнения – левую и правую части первого уравнения, умноженные на :

Теперь исключим y из третьего уравнения полученной системы уравнений:

Полученная СЛАУ равносильна системе .

Оставляем в левой части уравнений системы только слагаемые, содержащие неизвестные переменные x и y , а слагаемые с неизвестной переменной z переносим в правую часть:

Метод последовательного исключения неизвестных (метод Гаусса) задачи с решением

Метод последовательного исключения неизвестных (метод Гаусса)

Решение системы линейных уравнений методом Гаусса осуществляется по следующей схеме.

1. Выбираем одно из уравнений системы, в котором коэффициент при одном из неизвестных, например, при , отличен от нуля. Производя над уравнениями системы преобразования, которые приводят к равносильной системе, исключаем неизвестное из всех уравнений, кроме выбранного ранее. В этом заключается первый шаг метода Гаусса.

В результате первого шага может получиться такая система, о которой можно сразу сказать, что она несовместна, а следовательно, и данная система также несовместна.

Если полученная система состоит только из одного выбранного нами уравнения, то исходная система имеет одно решение или бесчисленное множество решений в зависимости от того, имеются ли свободные неизвестные. Во всех остальных случаях переходим ко второму шагу.

2. В системе, полученной в результате первого шага, выбираем одно из уравнений (отличное от выбранного в первом шаге), в котором коэффициент при другом неизвестном, например, при , отличен от нуля. Исключаем из всех уравнений, кроме двух выбранных.

Если это нужно, аналогично производим последующие шаги.

После нескольких шагов будет иметь место один из случаев:

а) получится явно несовместная система;

б) получится треугольная система, т. е. система вида

где , .

Система (7), а следовательно, и исходная система имеет единственное решение. Так как , то из последнего уравнения (7) находим . В предпоследнее уравнение подставляем , получим единственное значение для , так как . Продолжая этот процесс, находим последовательно . Указанный способ нахождения неизвестных называется обратным ходом метода Гаусса;

в) получится трапециевидная система, т. е. система вида:

где , , .

В системе (8) число неизвестных больше числа уравнений. Так как , то из последнего уравнения этой системы единственным образом выражается через . Осуществляя обратный ход, выразим единственным образом неизвестные через . Придавая последним произвольные значения , получим бесконечно много решений системы (8), а следовательно, и данной системы.

Заметим, что при применении метода Гаусса на практике имеет смысл вместо преобразований системы производить соответствующие преобразования над строками расширенной матрицы системы, т. е. приводить расширенную матрицу системы к трапециевидной с помощью элементарных преобразований над строками.

Задача №11.

Решить методом Гаусса систему

Решение:

Расширенная матрица системы имеет вид

Прибавив ко второй строке первую, умноженную на (-2), к третьей — первую, умноженную на (-3), к четвертой — первую, умноженную на (-1), получим

Разделим третью строку на 13 и поменяем местами вторую и третью строки:

Прибавим к третьей строке вторую, умноженную на (-9), к четвертой — вторую, умноженную на (-2):

Разделив вторую строку на (-2), а третью на (-7), имеем:

Этой матрице соответствует система

Осуществляя обратный ход, находим:

Таким образом, множеством решений будет

Задача №12.

Решить методом Гаусса систему уравнений

Решение:

Расширенная матрица системы имеет вид

Поменяв местами первую и вторую строки, имеем

Прибавив ко второй строке первую, умноженную на (-2), а к третьей — первую, умноженную на (-3), получим

Прибавив к третьей строке вторую, умноженную на (-1), получим

Этой матрице соответствует система

Осуществляя обратный ход, находим:

Этот материал взят со страницы кратких лекций с решением задач по высшей математике:

Решение задач по высшей математике

Возможно эти страницы вам будут полезны:

МЕТОД ГАУССА – Системы линейных уравнений

Ме́тод Га́усса – классический метод решения системы линейных алгебраических уравнений (СЛАУ). Это метод последовательного исключения переменных, когда с помощью элементарных преобразований система уравнений приводится к равносильной системе треугольного вида, из которой последовательно, начиная с последних (по номеру), находятся все переменные системы.

Хотя в настоящее время данный метод повсеместно называется методом Гаусса, он был известен и до К. Ф. Гаусса. Первое известное описание данного метода – в китайском трактате «Математика в девяти книгах».

Пусть исходная система выглядит следующим образом

Матрица A называется основной матрицей системы, – столбцом свободных членов.

Тогда, согласно свойству элементарных преобразований над строками, основную матрицу этой системы можно привести к ступенчатому виду (эти же преобразования нужно применять к столбцу свободных членов):

При этом будем считать, что базисный минор (ненулевой минор максимального порядка) основной матрицы находится в верхнем левом углу, то есть в него входят только коэффициенты при переменных .

Тогда переменные называются главными переменными. Все остальные называются свободными.

Если хотя бы одно число , где , то рассматриваемая система несовместна, т.е. у неё нет ни одного решения.

Пусть для любых .

Перенесём свободные переменные за знаки равенств и поделим каждое из уравнений системы на свой коэффициент при самом левом (, где   — номер строки):


где

Если свободным переменным системы (2) придавать все возможные значения и решать новую систему относительно главных неизвестных снизу вверх (то есть от нижнего уравнения к верхнему), то мы получим все решения этой СЛАУ. Так как эта система получена путём элементарных преобразований над исходной системой (1), то по теореме об эквивалентности при элементарных преобразованиях системы (1) и (2) эквивалентны, то есть множества их решений совпадают.

Условие совместности

Упомянутое выше условие для всех может быть сформулировано в качестве необходимого и достаточного условия совместности:

Напомним, что рангом совместной системы называется ранг её основной матрицы (либо расширенной, так как они равны).

Алгоритм решения СЛАУ методом Гаусса подразделяется на два этапа.

  • На первом этапе осуществляется так называемый прямой ход, когда путём элементарных преобразований над строками систему приводят к ступенчатой или треугольной форме, либо устанавливают, что система несовместна. А именно, среди элементов первого столбца матрицы выбирают ненулевой, перемещают его на крайнее верхнее положение перестановкой строк и вычитают получившуюся после перестановки первую строку из остальных строк, домножив её на величину, равную отношению первого элемента каждой из этих строк к первому элементу первой строки, обнуляя тем самым столбец под ним. После того, как указанные преобразования были совершены, первую строку и первый столбец мысленно вычёркивают и продолжают пока не останется матрица нулевого размера. Если на какой-то из итераций среди элементов первого столбца не нашёлся ненулевой, то переходят к следующему столбцу и проделывают аналогичную операцию.
  • На втором этапе осуществляется так называемый обратный ход, суть которого заключается в том, чтобы выразить все получившиеся базисные переменные через небазисные и построить фундаментальную систему решений, либо, если все переменные являются базисными, то выразить в численном виде единственное решение системы линейных уравнений. Эта процедура начинается с последнего уравнения, из которого выражают соответствующую базисную переменную (а она там всего одна) и подставляют в предыдущие уравнения, и так далее, поднимаясь по «ступенькам» наверх. Каждой строчке соответствует ровно одна базисная переменная, поэтому на каждом шаге, кроме последнего (самого верхнего), ситуация в точности повторяет случай последней строки.
Метод Гаусса требует арифметических операций.

В простейшем случае алгоритм выглядит так:

  • Обратный ход. Из последнего ненулевого уравнения выражаем базисную переменную через небазисные и подставляем в предыдущие уравнения. Повторяя эту процедуру для всех базисных переменных, получаем фундаментальное решение.

Пример 1


Покажем, как методом Гаусса можно решить следующую систему:

Обнулим коэффициенты при во второй и третьей строчках. Для этого прибавим к ним первую строчку, умноженную на и 1, соответственно:

Теперь обнулим коэффициент при  в третьей строке, вычтя из неё вторую строку, умноженную на 4:

В результате мы привели исходную систему к треугольному виду, тем самым закончим первый этап алгоритма.

На втором этапе разрешим полученные уравнения в обратном порядке. Имеем:

из третьего;

из второго, подставив полученное

из первого, подставив полученные и .

Таким образом исходная система решена.

В случае, если число уравнений в совместной системе получилось меньше числа неизвестных, то тогда ответ будет записываться в виде фундаментальной системы решений.

1.6 Метод Гаусса (метод последовательного исключения неизвестных)

Большим недостатком двух рассмотренных методов (формулы Крамера и матричный метод) является их большая трудоемкость, связанная с громоздкими вычислениями при большом значении . Возможность применения этих методов ограничена также размерностью системы уравненийи требованием невырожденности матрицы системы.

Обойти эти сложности помогает универсальный метод решения СЛАУ – метод Гаусса, который еще называют методом последовательного исключения неизвестных.

Условно в методе Гаусса можно выделить 2 этапа: «прямой» и «обратный» ходы.

Прямой ход предполагает последовательное исключение неизвестных из уравнений системы, начиная с 1-го (движение сверху вниз).

Обратный ход – последовательное нахождение значений неизвестных, начиная с последнего уравнения системы (движение снизу вверх).

В основу метода Гаусса положены свойства равносильных (эквивалентных) систем (т.е. имеющих одни и те же решения).

Примем без доказательства тот факт, что система переходит в ей равносильную в результате элементарных преобразований, к которым относятся:

– перестановка уравнений местами;

– умножение всех членов уравнения на одно и то же число, отличное от нуля;

– прибавление к обеим частям одного уравнения соответствующих частей другого уравнения, умноженных на одно и то же число.

Таким образом, суть Метода Гаусса заключается в том, что с помощью элементарных преобразований СЛАУ приводится к равносильной системе ступенчатого вида, из которой затем последовательно находятся значения переменных, начиная с последнего уравнения.

Замечание. Преобразования по схеме Гаусса удобно проводить не с самими уравнениями, а с расширенной матрицей системы:

В этом случае элементарные преобразования производятся над строками. Тогда, прямой ход будет состоять в получении нулей под главной диагональю. Затем по расширенной матрице восстанавливают систему и обратным ходом последовательно находят значения переменных.

Алгоритм Метода Гаусса цикличен, т.е. одна и та же операция будет по очереди повторяться для каждой неизвестной. Условно его можно выразить фразой: «диагональный элемент не ноль, под ним нули».

Рассмотрим СЛАУ (случай ):

(6.1)

Пусть (еслито перестановкой уравнений местами добьемся, чтобы выполнялось условие. Это приведет к равносильной системе уравнений). Разделим 1-е уравнение системы на. Оно примет вид:

где

ШАГ 1. Умножая это уравнение последовательно на и прибавляя полученные уравнения в том же порядке ко 2-му, 3-му … -му уравнениям, исключим переменную из всех уравнений системы, начиная со второго.

Получим:

(6.2)

ШАГ 2. Пусть (если это не так, то перестановкой уравнений или неизвестных с изменением их номеров добьемся, чтобы выполнялось условие).

Разделим второе уравнение системы на . Это уравнение примет вид

где

Умножая полученное уравнение последовательно на и прибавляя полученные уравнения в том же порядке к третьему … -му уравнению системы, исключим переменнуюиз всех уравнений системы, начиная с третьего.

Продолжая процесс последовательного исключения переменных после-го шага получим систему вида

(6.3)

Число нуль в последних уравнениях означает, что их левые части имеют вид . Если в полученной системе уравнений хотя бы одно из чиселне равно нулю, то соответствующее равенство противоречиво, и система (6.1) несовместна.

Для любой совместной системы (6.1) числа в системе (6.3) равны нулю. В этом случае последниеуравнений являются тождествами и их можно исключить при решении системы (6.3). После исключения из системы (6.3) последнихуравнений возможны два случая: 1) число уравнений, оставшихся в системе (6.3), равно числу неизвестных, т.е., тогда система (6.3) имеет треугольный вид и решение системы (6.1) единственное; 2)(тогда система (6.3) имеет ступенчатый вид и система (6.1) неопределенная, имеет бесчисленное множество решений).

Пример 1. Решить систему уравнений методом Гаусса:

Составим расширенную матрицу системы

Разделим первую строку матрицы на 2.

ШАГ 1.

ШАГ 2.

Разделим вторую строку полученной матрицы на

ШАГ 3.

Разделим третью строку на

Прямой ход на этом окончен.

Последней матрице соответствует система уравнений:

Обратный ход: из третьего уравнения системы

из второго уравнения системы

из первого уравнения системы

Ответ:

Пример 2. Методом Гаусса решить систему уравнений:

Запишем и преобразуем расширенную матрицу системы

Уравнение, соответствующее третьей строке полученной матрицы, представляет равенство . Следовательно, система несовместна.

Ответ: решений нет.

Замечания:

  1. Кроме решения СЛАУ методом Гаусса можно вычислять определители. После выполнения прямого хода определитель равен произведению элементов, стоящих на главной диагонали.

  2. Метод Гаусса помогает также находить обратную матрицу через присоединенную: к исходной матрицеприписывается рядом единичная; затем проводятся элементарные преобразования, приводящие матрицук виду единичной, при этом единичная матрица приводится к обратной

  3. При решении СЛАУ методом Гаусса одновременно осуществляется исследование системы.

Решение линейных систем уравнений методом Гаусса Метод исключения неизвестных

3. 4 Решение линейных систем уравнений методом Гаусса
Метод Гаусса – метод исключения неизвестных, когда несколько уравнений комбинируются таким образом, чтобы какое-то неизвестное сократилось. Но применить эти методы обычно можно после соответствующей подготовки системы.

Какие же преобразования возможно без риска приобрести лишние корни или, ещё хуже, потерять нужные решения?


  1. Любое уравнение система можно подвергнуть преобразованию, которое переводит его в равносильное уравнение: например, прибавить к обеим частям уравнения одно и то же число или умножить обе части на одно и то же ненулевое число.

  2. К системе можно приписать любое её следствие. И наоборот, если какое-либо уравнение системы является следствием остальных, его можно вычеркнуть.

  3. Любое уравнение системы можно заменить его на его сумму с любым другим уравнением, умноженным на произвольное число. Это преобразование основное в методе Гаусса.

  4. При соответствующих оговорках уравнения можно перемножать.

Ограничимся этими правилами, хотя существуют и другие.

Посмотрим, как действует это правило на конкретных примерах.

Пример. Решить систему

х + 2у = 4,

3х + 4у = 12.

Решение:

 Первую строку переписываем. Чтобы получить вторую, нужно умножить первую строку на 3 и из неё вычесть вторую. Получаем . Делим вторую строку на 2. . Тогда у = 0. Чтобы получить х, подставляем в первую строку: х = 4 – 2 . 0 = 4.

Ответ: (4; 0).

Пример: Решить методом обратной матрицы

Решение:

 Первую и вторую строку переписываем. Из третьей вычитаем первую и записываем на место третьей. Получаем:

 Первую и вторую строку переписываем. Вторую строку умножаем на 2 и вычитаем из третьей. Записываем на место третьей.

Тогда z = 3, у = 5 – 3 = 2, х = 4 – 3 – 0 = 1

Ответ: (1; 2; 3).

Пример: Решить систему

Решение:

 Первую и вторую строку переписываем. Из первой вычитаем третью и записываем на место третьей. Из первой вычитаем четвертую и записываем на место четвертой. Получаем:

 Первую и вторую строку переписываем. Из второй вычитаем третью и записываем на место третьей. Четвертую переписываем.

 Первую, вторую и третью строки переписываем. Из третьей вычитаем четвертую и записываем на место четвертой.

 Последнюю строку делим на 3. Получаем t = 4.

Возвращаясь последовательно на строку выше, находим: z = 11 – 2 . 4 = 3,

у = 9 – 3 – 4 = 2; х = 6 – 0 – 3 – 2 = 1.

Ответ: (1; 2; 3; 4)
Данное решение, на наш взгляд, гораздо проще, чем решение методом Крамера. Следующий пример рассмотрим менее подробно (не будем описывать простейшие преобразования).
Пример: Решить систему

Решение:

  

  t = 3; v = -3; z = -9; y = – 9; x = 4.

Ответ: (4; -9; -9; -3; 3)

3.5 Карл Фридрих Гаусс


ГАУСС (Gau) Карл Фридрих (1777-1855), немецкий математик, иностранный член-корреспондент (1802) и иностранный почетный член (1824) Петербургской АН. Родился 30 апреля 1777, Брауншвейг, ныне Германия, умер – 23 февраля 1855, Геттинген, Ганноверское королевство, ныне Германия.

Еще при жизни Гаусс был удостоен почетного титула «принц математиков», заслужил звание – Юный гений. Он был единственным сыном бедных родителей. Школьные учителя были так поражены его математическими и лингвистическими способностями, что обратились к герцогу Брауншвейгскому с просьбой о поддержке, и герцог дал деньги на продолжение обучения в школе и в Геттингенском университете (в 1795-98). Степень доктора Гаусс получил в 1799 в университете Хельмштедта.

Первое же обширное сочинение Гаусса «Арифметические исследования» (опубл.1801) на многие годы определило последующее развитие двух важных разделов математики — теории чисел и высшей алгебры. Из множества важных и тонких результатов, приведенных в «Арифметических исследованиях», следует отметить подробную теорию квадратичных форм и первое доказательство квадратичного закона взаимности. В конце сочинения Гаусс приводит полную теорию уравнений деления круга и, указывая их связь с задачей построения правильных многоугольников, решает стоявшую с античных времен проблему о возможности построения циркулем и линейкой правильного многоугольника с заданным числом сторон. Гаусс указал все числа, при которых построение правильного многоугольника с помощью циркуля и линейки возможно. Это пять так называемых гауссовых простых чисел: 3, 5, 17, 257 и 65337, а также умноженные на любую степень двойки произведения различных (не повторяющихся) гауссовых чисел. Например, построить с помощью циркуля и линейки правильный (3х5х17)-угольник можно, а правильный 7-угольник нельзя, так как семерка не гауссово простое число. Разумеется, доказанный Гауссом результат — пример так называемой чистой теоремы существования; утверждается, что построить с помощью циркуля и линейки правильный многоугольник с «допустимым» числом сторон можно, но ничего не говорится о том, как это сделать. Гаусс предложил также явный способ построения с помощью циркуля и линейки правильного 17-угольника. Это событие Гаусс посчитал столь значительным, что отметил его в «Дневнике» (запись от 30 марта 1796 г.) и завещал высечь правильный 17-угольник на своем надгробии (воля Гаусса была исполнена).

С именем Гаусса также связана основная теорема алгебры, согласно которой число корней многочлена (действительных и комплексных) равно степени многочлена (при подсчете числа корней кратный корень учитывается столько раз, какова его степень). Первое доказательство основной теоремы алгебры Гаусс дал в 1799, а позднее предложил еще несколько доказательств.

Гаусс живо интересовался не только «чистой математикой», но и ее приложениями. В области прикладной математики он не только получил ряд важных результатов, но и создал новые направления в науке.

Занимая с 1807 кафедру математики и астрономии Геттингенского университета и возглавляя астрономическую обсерваторию того же университета, Гаусс на протяжении более двух десятилетий занимается изучением орбит малых планет и их возмущений. Мировую известность обрел разработанный Гауссом метод определения эллиптической орбиты по трем наблюдениям. Применение этого метода к малой планете Церера позволило вновь найти ее на небе после того, как она была утеряна вскоре после ее открытия астрономом Дж. Пиацци (1801). Не меньший успех сопутствовал применению метода Гаусса к другой малой планете, Палладе (1802).

В 1809 выходит фундаментальный труд Гаусса «Теория движения небесных тел», в котором изложены методы вычисления планетных орбит, используемые (с незначительными усовершенствованиями) и поныне.

В 1812 Гаусс познакомил математический мир со своей гипергеометрической функцией, частным случаем которой являются многие из так называемых специальных функций математической физики. В той же работе он рассматривает и вопросы сходимости бесконечных рядов, важные для астрономических вычислений.

В 1818 Гаусс одним из первых начинает размышлять над созданием неевклидовой геометрии, но от публикации полученных результатов воздерживается, опасаясь, по собственному признанию, «криков беотийцев» (т.е. возражений и насмешек невежд).

Десятилетие 1820-30 застает Гаусса за проведением геодезической съемки Ганноверского королевства и составлением его подробной карты. Гаусс не только проделывает огромную организационную работу и руководит измерением длины дуги меридиана от Геттингена до Альтоны, но и создает основы «высшей геодезии», занимающейся описанием действительной формы земной поверхности. Обобщающий труд «Исследования о предметах высшей геодезии» Гаусс создает в 1842-47. В основе этого фундаментального труда лежат также принадлежащие Гауссу идеи так называемой внутренней геометрии поверхности, изложенной им в сочинении «Общие исследования о кривых поверхностях» (1827). Локальные (т. е. характеризующие малую окрестность точки) свойства поверхности, по мысли Гаусса, естественнее связывать не с «посторонними», введенными извне, а с внутренними криволинейными координатами и выражать через дифференциальную форму от внутренних координат. Если поверхность изгибать не растягивая, то ее внутренние свойства остаются неизменными. Впоследствии по образу и подобию внутренней геометрии поверхностей Гаусса была создана многомерная риманова геометрия.

Непреходящее значение для всех наук, имеющих дело с обработкой наблюдений, имеют разработанные Гауссом методы получения наиболее вероятных значений измеряемых величин. Особенно широкую известность получил созданный Гауссом в 1821-23 гг. метод наименьших квадратов. Гауссом заложены также и основы теории ошибок.

В 1830-40 гг. Гаусс много внимания уделяет проблемам физики. В 1832 он создает так называемую абсолютную систему единиц, приняв за основные три единицы; единицу времени 1 с, единицу длины 1 мм и единицу массы 1 м. В 1833 в тесном сотрудничестве с Вильгельмом Вебером Гаусс строит первый в Германии электромагнитный телеграф. В 1839 выходит сочинение Гаусса «Общая теория сил притяжения и отталкивания, действующих обратно пропорционально квадрату расстояния», в которой излагает основные положения теории потенциала и доказывает знаменитую теорему Гаусса—Остроградского. Работа «Диоптрические исследования» (1840) Гаусса посвящена теории построения изображений в сложных оптических системах.

Для творчества Гаусса характерна органическая связь между теоретической и прикладной математикой, широта проблематики. Труды Гаусса оказали большое влияние на развитие алгебры (доказательство основной теоремы алгебры), теории чисел (квадратичные вычеты), дифференциальной геометрии (внутренняя геометрия поверхностей), математической физики (принцип Гаусса), теории электричества и магнетизма, геодезии (разработка метода наименьших квадратов) и многих разделов астрономии.

Многие исследования Гаусс не публиковал при жизни. Они сохранились в виде очерков, набросков, переписки с друзьями. Изучением этих трудов до Второй мировой войны занималось Геттингенское научное общество, которому удалось издать 12 томов сочинений Гаусса. Наиболее интересную часть наследия составляет уже упоминавшийся дневник.

Научное творчество Гаусса наглядно показывает неосновательность деления наук на «чистые» и «прикладные»: «принц математиков» находил практические применения результатам своих фундаментальных исследований и из конкретных задач прикладных областей умел извлекать проблемы, представляющие интерес для фундаментальной науки.

Гауссовское исключение | Колледж алгебры

Результаты обучения

  • Напишите расширенную матрицу для системы уравнений.
  • Выполнение операций со строками в расширенной матрице.
  • Используйте метод исключения Гаусса для решения системы уравнений, представленной в виде расширенной матрицы.
  • Интерпретировать решение системы уравнений, представленной в виде расширенной матрицы.

Карл Фридрих Гаусс жил в конце 18-го и начале 19-го веков, но до сих пор считается одним из самых плодовитых математиков в истории.Его вклад в математику и физику охватывает такие области, как алгебра, теория чисел, анализ, дифференциальная геометрия, астрономия и оптика. Его открытия в области теории матриц изменили способ работы математиков за последние два столетия.

Немецкий математик Карл Фридрих Гаусс (1777–1855).

Мы впервые столкнулись с методом исключения Гаусса в «Системах линейных уравнений: две переменные». В этом разделе мы еще раз вернемся к этой технике решения систем, на этот раз с использованием матриц.

Операции со строками и расширенная матрица

Матрица может служить устройством для представления и решения системы уравнений. Чтобы выразить систему в матричной форме, мы извлекаем коэффициенты переменных и констант, и они становятся элементами матрицы. Мы используем вертикальную линию, чтобы отделить записи коэффициентов от констант, по сути заменяя знаки равенства. Когда система написана в такой форме, мы называем ее расширенной матрицей .

Например, рассмотрим следующую систему уравнений [латекс] 2 \ times 2 [/ латекс].

[латекс] \ begin {array} {l} 3x + 4y = 7 \\ 4x – 2y = 5 \ end {array} [/ latex]

Мы можем записать эту систему в виде расширенной матрицы:

[латекс] \ left [\ begin {array} {rr} \ hfill 3 & \ hfill 4 \\ \ hfill 4 & \ hfill -2 \ end {array} \ text {} | \ text {} \ begin {array} { r} \ hfill 7 \\ \ hfill 5 \ end {array} \ right] [/ latex]

Мы также можем написать матрицу, содержащую только коэффициенты. Это называется матрицей коэффициентов .

[латекс] \ left [\ begin {array} {cc} 3 & 4 \\ 4 & -2 \ end {array} \ right] [/ latex]

Трехмерная система уравнений , например

[латекс] \ begin {array} {l} 3x-yz = 0 \ hfill \\ \ text {} x + y = 5 \ hfill \\ \ text {} 2x – 3z = 2 \ hfill \ end {array} [/ латекс]

имеет матрицу коэффициентов

[латекс] \ left [\ begin {array} {rrr} \ hfill 3 & \ hfill -1 & \ hfill -1 \\ \ hfill 1 & \ hfill 1 & \ hfill 0 \\ \ hfill 2 & \ hfill 0 & \ hfill -3 \ конец {array} \ right] [/ latex]

и представлена ​​расширенной матрицей

[латекс] \ left [\ begin {array} {rrr} \ hfill 3 & \ hfill -1 & \ hfill -1 \\ \ hfill 1 & \ hfill 1 & \ hfill 0 \\ \ hfill 2 & \ hfill 0 & \ hfill -3 \ end {array} \ text {} | \ text {} \ begin {array} {r} \ hfill 0 \\ \ hfill 5 \\ \ hfill 2 \ end {array} \ right] [/ latex]

Обратите внимание, что матрица написана так, что переменные выстраиваются в свои собственные столбцы: термины x идут в первом столбце, y -термы во втором столбце и z -термы в третьем столбце.Очень важно, чтобы каждое уравнение было записано в стандартной форме [latex] ax + by + cz = d [/ latex], чтобы переменные совпадали. Если в уравнении отсутствует член переменной, коэффициент равен 0.

Как: по системе уравнений написать расширенную матрицу

  1. Запишите коэффициенты членов x в виде чисел в первом столбце.
  2. Запишите коэффициенты терминов и в виде чисел во втором столбце.
  3. Если есть z -термов, запишите коэффициенты в виде чисел в третьем столбце.
  4. Нарисуйте вертикальную линию и напишите константы справа от нее.

Пример: написание расширенной матрицы для системы уравнений

Напишите расширенную матрицу для данной системы уравнений.

[латекс] \ begin {array} {l} \ text {} x + 2y-z = 3 \ hfill \\ \ text {} 2x-y + 2z = 6 \ hfill \\ \ text {} x – 3y + 3z = 4 \ hfill \ end {array} [/ latex]

Показать решение

Расширенная матрица отображает коэффициенты переменных и дополнительный столбец для констант.

[латекс] \ left [\ begin {array} {rrr} \ hfill 1 & \ hfill 2 & \ hfill -1 \\ \ hfill 2 & \ hfill -1 & \ hfill 2 \\ \ hfill 1 & \ hfill -3 & \ hfill 3 \ end {array} \ text {} | \ text {} \ begin {array} {r} \ hfill 3 \\ \ hfill 6 \\ \ hfill 4 \ end {array} \ right] [/ latex]

Попробуйте

Напишите расширенную матрицу данной системы уравнений.

[латекс] \ begin {array} {l} 4x – 3y = 11 \\ 3x + 2y = 4 \ end {array} [/ latex]

Показать решение

[латекс] \ left [\ begin {array} {rr} \ hfill 4 & \ hfill -3 \\ \ hfill 3 & \ hfill 2 \ end {array} \ text {} | \ text {} \ begin {array} { r} \ hfill 11 \\ \ hfill 4 \ end {array} \ right] [/ latex]

Написание системы уравнений из расширенной матрицы

Мы можем использовать расширенные матрицы, чтобы помочь нам решать системы уравнений, потому что они упрощают операции, когда системы не перегружены переменными.Однако важно понимать, как переключаться между форматами, чтобы поиск решений был более плавным и интуитивно понятным. Здесь мы будем использовать информацию в расширенной матрице, чтобы записать систему уравнений в стандартной форме.

Пример: написание системы уравнений из расширенной матричной формы

Найдите систему уравнений из расширенной матрицы.

[латекс] \ left [\ begin {array} {rrr} \ hfill 1 & \ hfill -3 & \ hfill -5 \\ \ hfill 2 & \ hfill -5 & \ hfill -4 \\ \ hfill -3 & \ hfill 5 & \ hfill 4 \ end {array} \ text {} | \ text {} \ begin {array} {r} \ hfill -2 \\ \ hfill 5 \\ \ hfill 6 \ end {array} \ right] [/ latex]

Показать решение Когда столбцы представляют переменные [latex] x [/ latex], [latex] y [/ latex] и [latex] z [/ latex],

[латекс] \ left [\ begin {array} {rrr} \ hfill 1 & \ hfill -3 & \ hfill -5 \\ \ hfill 2 & \ hfill -5 & \ hfill -4 \\ \ hfill -3 & \ hfill 5 & \ hfill 4 \ end {array} \ text {} | \ text {} \ begin {array} {r} \ hfill -2 \\ \ hfill 5 \\ \ hfill 6 \ end {array} \ right] \ to \ begin { array} {l} x – 3y – 5z = -2 \ hfill \\ 2x – 5y – 4z = 5 \ hfill \\ -3x + 5y + 4z = 6 \ hfill \ end {array} [/ latex]

Попробуйте

Напишите систему уравнений из расширенной матрицы.

[латекс] \ left [\ begin {array} {ccc} 1 & -1 & 1 \\ 2 & -1 & 3 \\ 0 & 1 & 1 \ end {array} | \ begin {array} {c} 5 \\ 1 \\ -9 \ end {array} \ right] [/ latex]

Показать решение

[латекс] \ begin {array} {c} x-y + z = 5 \\ 2x-y + 3z = 1 \\ y + z = -9 \ end {array} [/ latex]

Операции со строками

Теперь, когда мы можем записывать системы уравнений в форме расширенной матрицы, мы рассмотрим различные операции со строками , которые могут выполняться с матрицей, такие как сложение, умножение на константу и перестановка строк.

Выполнение операций со строками в матрице – это метод, который мы используем для решения системы уравнений. Чтобы решить систему уравнений, мы хотим преобразовать матрицу в строковую форму , в которой есть единицы вниз по главной диагонали от верхнего левого угла до нижнего правого угла и нули в каждой позиции ниже. главная диагональ, как показано.

[латекс] \ begin {array} {c} \ text {Форма эшелона строк} \\ \ left [\ begin {array} {ccc} 1 & a & b \\ 0 & 1 & d \\ 0 & 0 & 1 \ end {массив } \ right] \ end {array} [/ latex]

Мы используем операции со строками, соответствующие операциям с уравнениями, чтобы получить новую матрицу, эквивалентную строке в более простой форме.Вот рекомендации по получению формы рядного эшелона.

  1. В любой ненулевой строке первым ненулевым числом является 1. Оно называется ведущим 1.
  2. Любые нулевые строки помещаются внизу матрицы.
  3. Любая ведущая 1 находится ниже и правее предыдущей ведущей 1.
  4. Любой столбец, в котором в начале стоит 1, имеет нули во всех остальных позициях в столбце.

Чтобы решить систему уравнений, мы можем выполнить следующие операции со строками, чтобы преобразовать матрицу коэффициентов в строковую форму и выполнить обратную подстановку, чтобы найти решение.

  1. Поменять местами ряды. (Обозначение: [латекс] {R} _ {i} \ leftrightarrow {R} _ {j} [/ latex])
  2. Умножьте строку на константу. (Обозначение: [латекс] c {R} _ {i} [/ latex])
  3. Добавить произведение одной строки на константу к другой строке. (Обозначение: [латекс] {R} _ {i} + c {R} _ {j} [/ latex])

Каждая из строковых операций соответствует операциям, которые мы уже научились решать системы уравнений с тремя переменными. С помощью этих операций есть несколько ключевых шагов, которые быстро достигнут цели написания матрицы в виде эшелона строк.Чтобы получить матрицу в виде эшелона строк для поиска решений, мы используем метод исключения Гаусса, который использует операции со строками для получения 1 в качестве первой записи, так что строку 1 можно использовать для преобразования оставшихся строк.

Общее примечание: исключение по Гауссу

Метод исключения Гаусса относится к стратегии, используемой для получения матрицы в виде строки-эшелона. Цель состоит в том, чтобы записать матрицу [latex] A [/ latex] с номером 1 в качестве записи вниз по главной диагонали и иметь все нули внизу.

[латекс] A = \ left [\ begin {array} {rrr} \ hfill {a} _ {11} & \ hfill {a} _ {12} & \ hfill {a} _ {13} \\ \ hfill {a} _ {21} & \ hfill {a} _ {22} & \ hfill {a} _ {23} \\ \ hfill {a} _ {31} & \ hfill {a} _ {32} & \ hfill {a} _ {33} \ end {array} \ right] \ stackrel {\ text {После исключения Гаусса}} {\ to} A = \ left [\ begin {array} {rrr} \ hfill 1 & \ hfill { b} _ {12} & \ hfill {b} _ {13} \\ \ hfill 0 & \ hfill 1 & \ hfill {b} _ {23} \\ \ hfill 0 & \ hfill 0 & \ hfill 1 \ end {array} \ справа] [/ латекс]

Первый шаг стратегии Гаусса включает получение 1 в качестве первой записи, так что строка 1 может использоваться для изменения строк ниже.

Как сделать: с учетом расширенной матрицы выполните операции со строками для получения формы «строка-эшелон»

  1. Первое уравнение должно иметь старший коэффициент 1. Поменяйте местами строки или умножьте на константу, если необходимо.
  2. Используйте операции со строками, чтобы получить нули в первом столбце под первой записью 1.
  3. Используйте операции со строками, чтобы получить 1 в строке 2, столбец 2.
  4. Используйте операции со строками, чтобы получить нули в нижнем столбце 2, под записью 1.
  5. Используйте операции со строками, чтобы получить 1 в строке 3, столбце 3.
  6. Продолжайте этот процесс для всех строк, пока в каждой записи по главной диагонали не будет 1, а внизу не останутся только нули.
  7. Если какие-либо строки содержат все нули, поместите их внизу.

Пример: выполнение операций со строками в расширенной матрице 3 × 3 для получения формы «Ряд-эшелон»

Выполнить операции со строками с заданной матрицей, чтобы получить форму строки-эшелона.

[латекс] \ left [\ begin {array} {rrr} \ hfill 1 & \ hfill -3 & \ hfill 4 \\ \ hfill 2 & \ hfill -5 & \ hfill 6 \\ \ hfill -3 & \ hfill 3 & \ hfill 4 \ end {array} \ text {} | \ text {} \ begin {array} {r} \ hfill 3 \\ \ hfill 6 \\ \ hfill 6 \ end {array} \ right] [/ latex]

Показать решение

В первой строке уже есть 1 в строке 1, столбце 1.Следующим шагом будет умножение строки 1 на [latex] -2 [/ latex] и добавление ее к строке 2. Затем замените строку 2 на результат.

[латекс] -2 {R} _ {1} + {R} _ {2} = {R} _ {2} \ to \ left [\ begin {array} {rrrrrr} \ hfill 1 & \ hfill & \ hfill -3 & \ hfill & \ hfill 4 & \ hfill \\ \ hfill 0 & \ hfill & \ hfill 1 & \ hfill & \ hfill -2 & \ hfill \\ \ hfill -3 & \ hfill & \ hfill 3 & \ hfill & \ hfill 4 & \ hfill \ end {array} | \ begin {array} {rr} \ hfill & \ hfill 3 \\ \ hfill & \ hfill 0 \\ \ hfill & \ hfill 6 \ end {array} \ right] [/ latex]

Затем получить ноль в строке 3, столбце 1.

[латекс] 3 {R} _ {1} + {R} _ {3} = {R} _ {3} \ to \ left [\ begin {array} {rrrrrr} \ hfill 1 & \ hfill & \ hfill – 3 & \ hfill & \ hfill 4 & \ hfill \\ \ hfill 0 & \ hfill & \ hfill 1 & \ hfill & \ hfill -2 & \ hfill \\ \ hfill 0 & \ hfill & \ hfill -6 & \ hfill & \ hfill 16 & \ hfill \ end {array} | \ begin {array} {rr} \ hfill & \ hfill 3 \\ \ hfill & \ hfill 0 \\ \ hfill & \ hfill 15 \ end {array} \ right] [/ latex]

Затем получить ноль в строке 3, столбце 2.

[латекс] 6 {R} _ {2} + {R} _ {3} = {R} _ {3} \ to \ left [\ begin {array} {rrrrrr} \ hfill 1 & \ hfill & \ hfill – 3 & \ hfill & \ hfill 4 & \ hfill \\ \ hfill 0 & \ hfill & \ hfill 1 & \ hfill & \ hfill -2 & \ hfill \\ \ hfill 0 & \ hfill & \ hfill 0 & \ hfill & \ hfill 4 & \ hfill \ end {array} | \ begin {array} {rr} \ hfill & \ hfill 3 \\ \ hfill & \ hfill 0 \\ \ hfill & \ hfill 15 \ end {array} \ right] [/ latex]

Последний шаг – получить 1 в строке 3, столбце 3.

[латекс] \ frac {1} {4} {R} _ {3} = {R} _ {3} \ to \ left [\ begin {array} {rrr} \ hfill 1 & \ hfill -3 & \ hfill 4 \\ \ hfill 0 & \ hfill 1 & \ hfill -2 \\ \ hfill 0 & \ hfill 0 & \ hfill 1 \ end {array} \ text {} | \ text {} \ begin {array} {r} \ hfill 3 \\ \ hfill -6 \\ \ hfill \ frac {15} {4} \ end {array} \ right] [/ latex]

Попробуйте

Запишите систему уравнений в виде строк.

[латекс] \ begin {array} {l} \ text {} x – 2y + 3z = 9 \ hfill \\ \ text {} -x + 3y = -4 \ hfill \\ 2x – 5y + 5z = 17 \ hfill \ end {array} [/ latex]

Показать решение

[латекс] \ left [\ begin {array} {ccc} 1 & – \ frac {5} {2} & \ frac {5} {2} \\ \ text {} 0 & 1 & 5 \\ 0 & 0 & 1 \ end {array} | \ begin {array} {c} \ frac {17} {2} \\ 9 \\ 2 \ end {array} \ right] [/ latex]

Решение системы с исключением Гаусса

Мы видели, как написать систему уравнений , с расширенной матрицей , а затем, как использовать операции со строками и обратную подстановку, чтобы получить форму эшелона строк .Теперь мы будем использовать метод исключения Гаусса как инструмент для решения системы, записанной в виде расширенной матрицы. В нашем первом примере мы покажем вам процесс использования исключения Гаусса в системе двух уравнений с двумя переменными.

Пример: решение системы 2 X 2 методом исключения Гаусса

Решите данную систему методом исключения Гаусса.

[латекс] \ begin {array} {l} 2x + 3y = 6 \ hfill \\ \ text {} x-y = \ frac {1} {2} \ hfill \ end {array} [/ latex]

Показать решение

Сначала мы запишем это как расширенную матрицу.

[латекс] \ left [\ begin {array} {rr} \ hfill 2 & \ hfill 3 \\ \ hfill 1 & \ hfill -1 \ end {array} \ text {} | \ text {} \ begin {array} { r} \ hfill 6 \\ \ hfill \ frac {1} {2} \ end {array} \ right] [/ latex]

Нам нужна 1 в строке 1, столбце 1. Этого можно добиться, поменяв местами строку 1 и строку 2.

[латекс] {R} _ {1} \ leftrightarrow {R} _ {2} \ to \ left [\ begin {array} {rrr} \ hfill 1 & \ hfill -1 & \ hfill \\ \ hfill 2 & \ hfill 3 & \ hfill \ end {array} | \ begin {array} {rr} \ hfill & \ hfill \ frac {1} {2} \\ \ hfill & \ hfill 6 \ end {array} \ right] [/ latex]

Теперь у нас есть 1 как первая запись в строке 1, столбце 1.Теперь давайте получим 0 в строке 2, столбце 1. Это можно сделать, умножив строку 1 на [latex] -2 [/ latex], а затем прибавив результат к строке 2.

[латекс] -2 {R} _ {1} + {R} _ {2} = {R} _ {2} \ to \ left [\ begin {array} {rrr} \ hfill 1 & \ hfill -1 & \ hfill \\ \ hfill 0 & \ hfill 5 & \ hfill \ end {array} | \ begin {array} {rr} \ hfill & \ hfill \ frac {1} {2} \\ \ hfill & \ hfill 5 \ end {массив } \ right] [/ latex]

У нас есть только один шаг, чтобы умножить строку 2 на [latex] \ frac {1} {5} [/ latex].

[латекс] \ frac {1} {5} {R} _ {2} = {R} _ {2} \ to \ left [\ begin {array} {rrr} \ hfill 1 & \ hfill -1 & \ hfill \ \ \ hfill 0 & \ hfill 1 & \ hfill \ end {array} | \ begin {array} {cc} & \ frac {1} {2} \\ & 1 \ end {array} \ right] [/ latex]

Использовать обратную замену.Вторая строка матрицы представляет [латекс] y = 1 [/ латекс]. Подставьте обратно [latex] y = 1 [/ latex] в первое уравнение.

[латекс] \ begin {array} {l} x- \ left (1 \ right) = \ frac {1} {2} \ hfill \\ \ text {} x = \ frac {3} {2} \ hfill \ end {array} [/ latex]

Решение – точка [латекс] \ left (\ frac {3} {2}, 1 \ right) [/ latex].

Попробуйте

Решите данную систему методом исключения Гаусса.

[латекс] \ begin {массив} {l} 4x + 3y = 11 \ hfill \\ \ text {} \ text {} \ text {} x – 3y = -1 \ hfill \ end {array} [/ latex]

Показать решение

[латекс] \ левый (2,1 \ правый) [/ латекс]

В нашем следующем примере мы решим систему двух уравнений с двумя зависимыми переменными.Напомним, что зависимая система имеет бесконечное количество решений, и результатом операций со строками в ее расширенной матрице будет уравнение, такое как [latex] 0 = 0 [/ latex]. Мы также рассмотрим написание общего решения для зависимой системы.

Пример: решение зависимой системы

Решите систему уравнений.

[латекс] \ begin {array} {l} 3x + 4y = 12 \\ 6x + 8y = 24 \ end {array} [/ latex]

Показать решение Выполните операций со строками на расширенной матрице, чтобы попытаться получить строковую форму .

[латекс] A = \ left [\ begin {array} {llll} 3 \ hfill & \ hfill & 4 \ hfill & \ hfill \\ 6 \ hfill & \ hfill & 8 \ hfill & \ hfill \ end {array} | \ begin {array} {ll} \ hfill & 12 \ hfill \\ \ hfill & 24 \ hfill \ end {array} \ right] [/ latex]

[латекс] \ begin {массив} {l} \ hfill \\ \ begin {array} {l} – \ frac {1} {2} {R} _ {2} + {R} _ {1} = { R} _ {1} \ to \ left [\ begin {array} {llll} 0 \ hfill & \ hfill & 0 \ hfill & \ hfill \\ 6 \ hfill & \ hfill & 8 \ hfill & \ hfill \ end { array} | \ begin {array} {ll} \ hfill & 0 \ hfill \\ \ hfill & 24 \ hfill \ end {array} \ right] \ hfill \\ {R} _ {1} \ leftrightarrow {R} _ {2} \ to \ left [\ begin {array} {llll} 6 \ hfill & \ hfill & 8 \ hfill & \ hfill \\ 0 \ hfill & \ hfill & 0 \ hfill & \ hfill \ end {array} | \ begin {array} {ll} \ hfill & 24 \ hfill \\ \ hfill & 0 \ hfill \ end {array} \ right] \ hfill \ end {array} \ hfill \ end {array} [/ latex]

Матрица заканчивается всеми нулями в последней строке: [latex] 0y = 0 [/ latex].Таким образом, существует бесконечное количество решений и система классифицируется как зависимая. Чтобы найти общее решение, вернитесь к одному из исходных уравнений и решите для [latex] y [/ latex].

[латекс] \ begin {array} {l} 3x + 4y = 12 \ hfill \\ \ text {} 4y = 12 – 3x \ hfill \\ \ text {} y = 3- \ frac {3} {4} x \ hfill \ end {array} [/ latex]

Итак, решение этой системы – [латекс] \ left (x, 3- \ frac {3} {4} x \ right) [/ latex].

Теперь мы перейдем на следующий шаг к решению системы линейных уравнений 3 на 3.Общая идея состоит в том, чтобы исключить все переменные, кроме одной, с помощью операций со строками, а затем выполнить обратную замену для поиска других переменных.

Пример: решение системы линейных уравнений с использованием матриц

Решите систему линейных уравнений с помощью матриц.

[латекс] \ begin {массив} {c} \ begin {array} {l} \ hfill \\ \ hfill \\ x-y + z = 8 \ hfill \ end {array} \\ 2x + 3y-z = -2 \\ 3x – 2y – 9z = 9 \ end {array} [/ latex]

Показать решение

Сначала мы пишем расширенную матрицу.

[латекс] \ left [\ begin {array} {rrr} \ hfill 1 & \ hfill -1 & \ hfill 1 \\ \ hfill 2 & \ hfill 3 & \ hfill -1 \\ \ hfill 3 & \ hfill -2 & \ hfill -9 \ end {array} \ text {} | \ text {} \ begin {array} {r} \ hfill 8 \\ \ hfill -2 \\ \ hfill 9 \ end {array} \ right] [/ latex]

Затем мы выполняем операции со строками, чтобы получить форму «строка-эшелон».

[латекс] \ begin {array} {rrrrr} \ hfill -2 {R} _ {1} + {R} _ {2} = {R} _ {2} \ to \ left [\ begin {array} { rrrrrr} \ hfill 1 & \ hfill & \ hfill -1 & \ hfill & \ hfill 1 & \ hfill \\ \ hfill 0 & \ hfill & \ hfill 5 & \ hfill & \ hfill -3 & \ hfill \\ \ hfill 3 & \ hfill & \ hfill -2 & \ hfill & \ hfill -9 & \ hfill \ end {array} | \ begin {array} {rr} \ hfill & \ hfill 8 \\ \ hfill & \ hfill -18 \\ \ hfill & \ hfill 9 \ end {массив} \ right] & \ hfill & \ hfill & \ hfill & \ hfill -3 {R} _ {1} + {R} _ {3} = {R} _ {3} \ to \ left [\ begin {array} {rrrrrr} \ hfill 1 & \ hfill & \ hfill -1 & \ hfill & \ hfill 1 & \ hfill \\ \ hfill 0 & \ hfill & \ hfill 5 & \ hfill & \ hfill -3 & \ hfill \\ \ hfill 0 & \ hfill & \ hfill 1 & \ hfill & \ hfill -12 & \ hfill \ end {array} | \ begin {array} {rr} \ hfill & \ hfill 8 \\ \ hfill & \ hfill -18 \\ \ hfill & \ hfill -15 \ end {array} \ right] \ end {array} [/ latex]

Самый простой способ получить 1 в строке 2 столбца 1 – это поменять местами [латекс] {R} _ {2} [/ latex] и [latex] {R} _ {3} [/ latex].

[латекс] \ text {Interchange} {R} _ {2} \ text {и} {R} _ {3} \ to \ left [\ begin {array} {rrrrrrr} \ hfill 1 & \ hfill & \ hfill – 1 & \ hfill & \ hfill 1 & \ hfill & \ hfill 8 \\ \ hfill 0 & \ hfill & \ hfill 1 & \ hfill & \ hfill -12 & \ hfill & \ hfill -15 \\ \ hfill 0 & \ hfill & \ hfill 5 & \ hfill & \ hfill -3 & \ hfill & \ hfill -18 \ end {array} \ right] [/ latex]

Затем

[латекс] \ begin {array} {l} \\ \ begin {array} {rrrrr} \ hfill -5 {R} _ {2} + {R} _ {3} = {R} _ {3} \ в \ left [\ begin {array} {rrrrrr} \ hfill 1 & \ hfill & \ hfill -1 & \ hfill & \ hfill 1 & \ hfill \\ \ hfill 0 & \ hfill & \ hfill 1 & \ hfill & \ hfill -12 & \ hfill \\ \ hfill 0 & \ hfill & \ hfill 0 & \ hfill & \ hfill 57 & \ hfill \ end {array} | \ begin {array} {rr} \ hfill & \ hfill 8 \\ \ hfill & \ hfill -15 \\ \ hfill & \ hfill 57 \ end {array} \ right] & \ hfill & \ hfill & \ hfill & \ hfill – \ frac {1} {57} {R} _ {3} = {R} _ {3} \ to \ left [\ begin {array} {rrrrrr} \ hfill 1 & \ hfill & \ hfill -1 & \ hfill & \ hfill 1 & \ hfill \\ \ hfill 0 & \ hfill & \ hfill 1 & \ hfill & \ hfill -12 & \ hfill \\ \ hfill 0 & \ hfill & \ hfill 0 & \ hfill & \ hfill 1 & \ hfill \ end {array} | \ begin {array} {rr} \ hfill & \ hfill 8 \\ \ hfill & \ hfill -15 \ \ \ hfill & \ hfill 1 \ end {array} \ right] \ end {array} \ end {array} [/ latex]

Последняя матрица представляет собой эквивалентную систему.

[латекс] \ begin {массив} {l} \ text {} x-y + z = 8 \ hfill \\ \ text {} y – 12z = -15 \ hfill \\ \ text {} z = 1 \ hfill \ end {array} [/ latex]

Используя обратную подстановку, мы получаем решение как [latex] \ left (4, -3,1 \ right) [/ latex].

Напомним, что есть три возможных исхода решений для линейных систем. В предыдущем примере решение [латекс] \ left (4, -3,1 \ right) [/ latex] представляет точку в трехмерном пространстве. Эта точка представляет собой пересечение трех плоскостей.В следующем примере мы решаем систему, используя операции со строками, и обнаруживаем, что она представляет зависимую систему. Зависимая система в 3-х измерениях может быть представлена ​​двумя идентичными плоскостями, как в 2-х измерениях, где зависимая система представляет две идентичные линии.

Пример: решение 3 x 3 зависимой системы

Решите следующую систему линейных уравнений, используя метод исключения Гаусса.

[латекс] \ begin {array} {r} \ hfill -x – 2y + z = -1 \\ \ hfill 2x + 3y = 2 \\ \ hfill y – 2z = 0 \ end {array} [/ latex]

Показать решение

Напишите расширенную матрицу.

[латекс] \ left [\ begin {array} {rrr} \ hfill -1 & \ hfill -2 & \ hfill 1 \\ \ hfill 2 & \ hfill 3 & \ hfill 0 \\ \ hfill 0 & \ hfill 1 & \ hfill -2 \ end {array} \ text {} | \ text {} \ begin {array} {r} \ hfill -1 \\ \ hfill 2 \\ \ hfill 0 \ end {array} \ right] [/ latex]

Сначала умножьте строку 1 на [latex] -1 [/ latex], чтобы получить 1 в строке 1, столбце 1. Затем выполните операции со строками , чтобы получить форму эшелона строки.

[латекс] – {R} _ {1} \ to \ left [\ begin {array} {rrrrrrr} \ hfill 1 & \ hfill & \ hfill 2 & \ hfill & \ hfill -1 & \ hfill & \ hfill 1 \\ \ hfill 2 & \ hfill & \ hfill 3 & \ hfill & \ hfill 0 & \ hfill & \ hfill 2 \\ \ hfill 0 & \ hfill & \ hfill 1 & \ hfill & \ hfill -2 & \ hfill & \ hfill 0 \ end {array} \ справа] [/ латекс]

[латекс] {R} _ {2} \ leftrightarrow {R} _ {3} \ to \ left [\ begin {array} {rrrrr} \ hfill 1 & \ hfill & \ hfill 2 & \ hfill & \ hfill -1 \ \ \ hfill 0 & \ hfill & \ hfill 1 & \ hfill & \ hfill -2 \\ \ hfill 2 & \ hfill & \ hfill 3 & \ hfill & \ hfill 0 \ end {array} \ text {} | \ begin {array} { rr} \ hfill & \ hfill 1 \\ \ hfill & \ hfill 0 \\ \ hfill & \ hfill 2 \ end {array} \ right] [/ latex]

[латекс] -2 {R} _ {1} + {R} _ {3} = {R} _ {3} \ to \ left [\ begin {array} {rrrrrr} \ hfill 1 & \ hfill & \ hfill 2 & \ hfill & \ hfill -1 & \ hfill \\ \ hfill 0 & \ hfill & \ hfill 1 & \ hfill & \ hfill -2 & \ hfill \\ \ hfill 0 & \ hfill & \ hfill -1 & \ hfill & \ hfill 2 & \ hfill \ end {array} | \ begin {array} {rr} \ hfill & \ hfill 1 \\ \ hfill & \ hfill 0 \\ \ hfill & \ hfill 0 \ end {array} \ right] [/ latex]

[латекс] {R} _ {2} + {R} _ {3} = {R} _ {3} \ to \ left [\ begin {array} {rrrrrr} \ hfill 1 & \ hfill & \ hfill 2 & \ hfill & \ hfill -1 & \ hfill \\ \ hfill 0 & \ hfill & \ hfill 1 & \ hfill & \ hfill -2 & \ hfill \\ \ hfill 0 & \ hfill & \ hfill 0 & \ hfill & \ hfill 0 & \ hfill \ end { array} | \ begin {array} {rr} \ hfill & \ hfill 2 \\ \ hfill & \ hfill 1 \\ \ hfill & \ hfill 0 \ end {array} \ right] [/ latex]

Последняя матрица представляет следующую систему.

[латекс] \ begin {array} {l} \ text {} x + 2y-z = 1 \ hfill \\ \ text {} y – 2z = 0 \ hfill \\ \ text {} 0 = 0 \ hfill \ конец {array} [/ latex]

По тождеству [latex] 0 = 0 [/ latex] мы видим, что это зависимая система с бесконечным числом решений. Затем мы находим общее решение. Решив второе уравнение для [латекс] y [/ латекс] и подставив его в первое уравнение, мы можем решить для [латекс] z [/ латекс] через [латекс] x [/ латекс].

[латекс] \ begin {array} {l} \ text {} x + 2y-z = 1 \ hfill \\ \ text {} y = 2z \ hfill \\ \ hfill \\ x + 2 \ left (2z \ справа) -z = 1 \ hfill \\ \ text {} x + 3z = 1 \ hfill \\ \ text {} z = \ frac {1-x} {3} \ hfill \ end {array} [/ latex]

Теперь мы подставляем выражение для [latex] z [/ latex] во второе уравнение, чтобы решить для [latex] y [/ latex] через [latex] x [/ latex].

[латекс] \ begin {массив} {l} \ text {} y – 2z = 0 \ hfill \\ \ text {} z = \ frac {1-x} {3} \ hfill \\ \ hfill \\ y – 2 \ left (\ frac {1-x} {3} \ right) = 0 \ hfill \\ \ text {} y = \ frac {2 – 2x} {3} \ hfill \ end {array} [/ latex ]

Общее решение: [latex] \ left (x, \ frac {2 – 2x} {3}, \ frac {1-x} {3} \ right) [/ latex].

Общее решение для зависимой системы 3 X 3

Напомним, что когда вы решаете зависимую систему линейных уравнений с двумя переменными с использованием исключения или подстановки, вы можете записать решение [latex] (x, y) [/ latex] через x, потому что существует бесконечно много (x, y) пары, которые будут удовлетворять зависимой системе уравнений, и все они попадают на линию [латекс] (x, mx + b) [/ latex].Теперь, когда вы работаете в трех измерениях, решение будет представлять собой плоскость, поэтому вы должны записать его в общей форме [латекс] (x, m_ {1} x + b_ {1}, m_ {2} x + b_ { 2}) [/ латекс].

Попробуйте

Решите систему, используя метод исключения Гаусса.

[латекс] \ begin {array} {c} x + 4y-z = 4 \\ 2x + 5y + 8z = 15 \ x + 3y – 3z = 1 \ end {array} [/ latex]

Показать решение

[латекс] \ левый (1,1,1 \ правый) [/ латекс]

Вопросы и ответы

Можно ли решить любую систему линейных уравнений методом исключения Гаусса?

Да, система линейных уравнений любого размера может быть решена методом исключения Гаусса.

Практическое руководство. Для данной системы уравнений решите с помощью матриц с помощью калькулятора.

  1. Сохраните расширенную матрицу как матричную переменную [latex] \ left [A \ right], \ left [B \ right], \ left [C \ right] \ text {,} \ dots [/ latex].
  2. Используйте в калькуляторе функцию ref (, вызывая каждую матричную переменную по мере необходимости.

Пример: решение систем уравнений с помощью калькулятора

Решите систему уравнений.

[латекс] \ begin {array} {r} \ hfill 5x + 3y + 9z = -1 \\ \ hfill -2x + 3y-z = -2 \\ \ hfill -x – 4y + 5z = 1 \ end { array} [/ latex]

Показать решение

Напишите расширенную матрицу для системы уравнений.

[латекс] \ left [\ begin {array} {rrr} \ hfill 5 & \ hfill 3 & \ hfill 9 \\ \ hfill -2 & \ hfill 3 & \ hfill -1 \\ \ hfill -1 & \ hfill -4 & \ hfill 5 \ end {array} \ text {} | \ text {} \ begin {array} {r} \ hfill -1 \\ \ hfill -2 \\ \ hfill 1 \ end {array} \ right] [/ latex]

На странице матриц калькулятора введите расширенную матрицу выше как матричную переменную [latex] \ left [A \ right] [/ latex].

[латекс] \ left [A \ right] = \ left [\ begin {array} {rrrrrrr} \ hfill 5 & \ hfill & \ hfill 3 & \ hfill & \ hfill 9 & \ hfill & \ hfill -1 \\ \ hfill – 2 & \ hfill & \ hfill 3 & \ hfill & \ hfill -1 & \ hfill & \ hfill -2 \\ \ hfill -1 & \ hfill & \ hfill -4 & \ hfill & \ hfill 5 & \ hfill & \ hfill 1 \ end {массив } \ right] [/ latex]

Используйте функцию ref ( в калькуляторе, вызывая матричную переменную [latex] \ left [A \ right] [/ latex].

[латекс] \ text {ref} \ left (\ left [A \ right] \ right) [/ latex]

Оценить.

[латекс] \ begin {array} {l} \ hfill \\ \ left [\ begin {array} {rrrr} \ hfill 1 & \ hfill \ frac {3} {5} & \ hfill \ frac {9} {5 } & \ hfill \ frac {1} {5} \\ \ hfill 0 & \ hfill 1 & \ hfill \ frac {13} {21} & \ hfill – \ frac {4} {7} \\ \ hfill 0 & \ hfill 0 & \ hfill 1 & \ hfill – \ frac {24} {187} \ end {array} \ right] \ to \ begin {array} {l} x + \ frac {3} {5} y + \ frac {9} {5} z = – \ frac {1} {5} \ hfill \\ \ text {} y + \ frac {13} {21} z = – \ frac {4} {7} \ hfill \\ \ text {} z = – \ frac {24} {187} \ hfill \ end {array} \ hfill \ end {array} [/ latex]

При использовании обратной подстановки решение: [latex] \ left (\ frac {61} {187}, – \ frac {92} {187}, – \ frac {24} {187} \ right) [/ latex] .

Приложения систем уравнений

Теперь обратимся к приложениям, для которых используются системы уравнений. В следующем примере мы определяем, сколько денег было инвестировано по двум разным ставкам, учитывая сумму процентов, полученных на обоих счетах.

Пример: применение матриц 2 × 2 к финансам

Кэролайн инвестирует в общей сложности 12 000 долларов в две муниципальные облигации, одна из которых выплачивает 10,5% годовых, а другая – 12%. Годовой процент, полученный по двум инвестициям в прошлом году, составил 1335 долларов.Сколько было вложено по каждой ставке?

Показать решение

У нас есть система двух уравнений с двумя переменными. Пусть [latex] x = [/ latex] сумма, инвестированная под 10,5% годовых, и [latex] y = [/ latex] сумма, инвестированная под 12% годовых.

[латекс] \ begin {массив} {l} \ text {} x + y = 12 000 \ hfill \\ 0,105x + 0,12y = 1,335 \ hfill \ end {array} [/ latex]

В качестве матрицы имеем

[латекс] \ left [\ begin {array} {rr} \ hfill 1 & \ hfill 1 \\ \ hfill 0.105 & \ hfill 0.12 \ end {array} \ text {} | \ text {} \ begin {array} { r} \ hfill 12 000 \\ \ hfill 1,335 \ end {array} \ right] [/ latex]

Умножить строку 1 на [латекс] -0.105 [/ latex] и добавьте результат в строку 2.

[латекс] \ left [\ begin {array} {rr} \ hfill 1 & \ hfill 1 \\ \ hfill 0 & \ hfill 0.015 \ end {array} \ text {} | \ text {} \ begin {array} {r } \ hfill 12,000 \\ \ hfill 75 \ end {array} \ right] [/ latex]

Затем,

[латекс] \ begin {array} {l} 0,015y = 75 \ hfill \\ \ text {} y = 5,000 \ hfill \ end {array} [/ latex]

Итак [латекс] 12 000 – 5 000 = 7 000 [/ латекс].

Таким образом, 5 000 долларов США были инвестированы под 12% годовых и 7 000 долларов США – под 10,5% годовых.

Пример: применение матриц 3 × 3 к финансам

Ava инвестирует в общей сложности 10 000 долларов в три счета, один из которых платит 5% годовых, другой – 8%, а третий – 9%.Годовой процент, полученный по трем инвестициям в прошлом году, составил 770 долларов. Сумма, вложенная под 9%, была вдвое больше, чем сумма, вложенная под 5%. Сколько было вложено по каждой ставке?

Показать решение

У нас есть система трех уравнений с тремя переменными. Пусть [latex] x [/ latex] будет сумма, инвестированная под 5% годовых, пусть [latex] y [/ latex] будет суммой, инвестированной под 8%, и пусть [latex] z [/ latex] будет инвестированной суммой. под 9% годовых. Таким образом,

[латекс] \ begin {array} {l} \ text {} x + y + z = 10 000 \ hfill \\ 0.05x + 0,08y + 0,09z = 770 \ hfill \\ \ text {} 2x-z = 0 \ hfill \ end {array} [/ latex]

В качестве матрицы имеем

[латекс] \ left [\ begin {array} {rrr} \ hfill 1 & \ hfill 1 & \ hfill 1 \\ \ hfill 0,05 & \ hfill 0,08 & \ hfill 0,09 \\ \ hfill 2 & \ hfill 0 & \ hfill -1 \ end {array} \ text {} | \ text {} \ begin {array} {r} \ hfill 10,000 \\ \ hfill 770 \\ \ hfill 0 \ end {array} \ right] [/ latex]

Теперь мы выполняем исключение Гаусса, чтобы получить форму строки-эшелон.

[латекс] \ begin {массив} {l} \ begin {array} {l} \ hfill \\ -0.05 {R} _ {1} + {R} _ {2} = {R} _ {2} \ to \ left [\ begin {array} {rrrrrr} \ hfill 1 & \ hfill & \ hfill 1 & \ hfill & \ hfill 1 & \ hfill \\ \ hfill 0 & \ hfill & \ hfill 0.03 & \ hfill & \ hfill 0.04 & \ hfill \\ \ hfill 2 & \ hfill & \ hfill 0 & \ hfill & \ hfill -1 & \ hfill \ end {array} | \ begin {array} {rr} \ hfill & \ hfill 10,000 \\ \ hfill & \ hfill 270 \\ \ hfill & \ hfill 0 \ end {array} \ right] \ hfill \ end {array} \ hfill \\ -2 {R} _ {1} + {R} _ {3} = {R} _ {3} \ to \ left [\ begin {array} {rrrrrr} \ hfill 1 & \ hfill & \ hfill 1 & \ hfill & \ hfill 1 & \ hfill \\ \ hfill 0 & \ hfill & \ hfill 0.03 & \ hfill & \ hfill 0.04 & \ hfill \\ \ hfill 0 & \ hfill & \ hfill -2 & \ hfill & \ hfill -3 & \ hfill \ end {array} | \ begin {array} {rr} \ hfill & \ hfill 10,000 \\ \ hfill & \ hfill 270 \\ \ hfill & \ hfill -20,000 \ end {array} \ right] \ hfill \\ \ frac {1} {0.03} {R} _ {2} = {R} _ {2} \ to \ left [\ begin {array} {rrrrrr} \ hfill 0 & \ hfill & \ hfill 1 & \ hfill & \ hfill 1 & \ hfill \\ \ hfill 0 & \ hfill & \ hfill 1 & \ hfill & \ hfill \ frac {4} {3} & \ hfill \\ \ hfill 0 & \ hfill & \ hfill -2 & \ hfill & \ hfill -3 & \ hfill \ end {array} | \ begin {array} {rr} \ hfill & \ hfill 10,000 \\ \ hfill & \ hfill 9,000 \\ \ hfill & \ hfill -20,000 \ end {array} \ right] \ hfill \\ 2 {R} _ {2} + {R} _ {3} = {R} _ {3} \ to \ left [\ begin {array} {rrrrrr} \ hfill 1 & \ hfill & \ hfill 1 & \ hfill & \ hfill 1 & \ hfill \\ \ hfill 0 & \ hfill & \ hfill 1 & \ hfill & \ hfill \ frac {4} {3} & \ hfill \\ \ hfill 0 & \ hfill & \ hfill 0 & \ hfill & \ hfill – \ frac {1} {3} & \ hfill \ end {array} | \ begin {array} {rr} \ hfill & \ hfill 10,000 \\ \ hfill & \ hfill 9,000 \\ \ hfill & \ hfill -2,000 \ end {array} \ right] \ hfill \ end {array} [/ latex]

Третья строка сообщает нам [латекс] – \ frac {1} {3} z = -2,000 [/ latex]; таким образом [латекс] z = 6,000 [/ латекс].

Вторая строка сообщает нам [латекс] y + \ frac {4} {3} z = 9000 [/ latex].

Подставляя [латекс] z = 6,000 [/ latex], получаем

[латекс] \ begin {array} {r} \ hfill y + \ frac {4} {3} \ left (6000 \ right) = 9000 \\ \ hfill y + 8000 = 9000 \\ \ hfill y = 1000 \ end {array} [/ latex]

Первая строка сообщает нам [латекс] x + y + z = 10,000 [/ latex]. Подставив [latex] y = 1,000 [/ latex] и [latex] z = 6,000 [/ latex], мы получим
[latex] \ begin {array} {l} x + 1,000 + 6,000 = 10,000 \ hfill \\ \ text {} x = 3 000 \ text {} \ hfill \ end {array} [/ latex]

Ответ: 3000 долларов вложены под 5%, 1000 долларов вложены под 8% и 6000 долларов вложены под 9%.

Попробуйте

Небольшая обувная компания взяла ссуду в размере 1 500 000 долларов на расширение своего ассортимента. Часть денег была взята под 7%, часть – под 8%, часть – под 10%. Сумма займа под 10% в четыре раза превышала сумму займа под 7%, а годовая процентная ставка по всем трем займам составляла 130 500 долларов. Используйте матрицы, чтобы найти сумму, заимствованную по каждой ставке.

Показать решение

150 000 долларов США под 7%, 750 000 долларов США под 8%, 600 000 долларов США под 10%

Ключевые понятия

  • Расширенная матрица – это матрица, которая содержит коэффициенты и константы системы уравнений.
  • Матрица, дополненная постоянным столбцом, может быть представлена ​​как исходная система уравнений.
  • Операции со строками включают в себя умножение строки на константу, добавление одной строки к другой строке и замену строк местами.
  • Мы можем использовать метод исключения Гаусса для решения системы уравнений.
  • Операции со строками выполняются над матрицами для получения формы «строка-эшелон».
  • Чтобы решить систему уравнений, запишите ее в форме расширенной матрицы. Выполните операции со строками, чтобы получить форму эшелона строк.Обратно-заменитель, чтобы найти решения.
  • Калькулятор можно использовать для решения систем уравнений с использованием матриц.
  • Многие реальные проблемы можно решить с помощью расширенных матриц.

Глоссарий

дополненная матрица
матрица коэффициентов, примыкающая к столбцу констант, разделенному вертикальной линией в скобках матрицы
матрица коэффициентов
матрица, содержащая только коэффициенты из системы уравнений
Исключение по Гауссу
с использованием элементарных операций со строками для получения матрицы в виде строки-эшелона
главная диагональ
записей из левого верхнего угла по диагонали в правый нижний угол квадратной матрицы
рядная форма
после выполнения строковых операций матричная форма, содержащая единицы по главной диагонали и нули в каждом пробеле ниже диагонали
эквивалент строки
две матрицы [latex] A [/ latex] и [latex] B [/ latex] эквивалентны строкам, если одна может быть получена из другой путем выполнения базовых операций со строками
рядные операции
добавление одной строки к другой, умножение строки на константу, перестановка строк и т. Д. С целью получения формы «строка-эшелон»

Метод исключения Гаусса

Идея метода исключения Гаусса заключается в следующем.Учитывая систему уравнений, мы используем правила предыдущего уровня для создания эквивалентной системы эшелонов, чтобы мы могли легко ее решить. Сам метод Гаусса – это процедура преобразования системы в эквивалентную эшелонированную систему.

Пусть система: $$$ \ left \ {\ begin {array} {c} 3x + 2y + z = 1 \\ 5x + 3y + 4z = 2 \ x + y-z = 1 \ end {array} \ right. $$$

Первый шаг – написание системы в матричной форме. Посмотрите, что в матрице мы берем коэффициенты и постоянные члены.$$$ \ begin {pmatrix} 3 & 2 & 1 & | 1 \\ 5 & 3 & 4 & | 2 \\ 1 & 1 & -1 & | 1 \ end {pmatrix} $$$

Используя уже известные правила, мы должны получить систему эшелонов, которая будет выглядеть следующим образом: $$$ \ begin {pmatrix} a_ {11} & a_ {12} & a_ {13} & | b_1 \\ \ fbox {} & a_ {22} & a_ {23} & | b_2 \\ \ fbox {} & \ fbox {} & a_ {33} & | b_3 \ end {pmatrix} $$$ Давай найдем это $$$ \ begin {pmatrix} 3 & 2 & 1 & | 1 \\ 5 & 3 & 4 & | 2 \\ 1 & 1 & -1 & | 1 \ end {pmatrix} \ rightarrow (r3 \ leftrightarrow r1) \правая стрелка \ begin {pmatrix} 1 & 1 & -1 & | 1 \\ 3 & 2 & 1 & | 1 \\ 5 & 3 & 4 & | 2 \ end {pmatrix} \ rightarrow \ left \ {\ begin {array} {c} r2-3r1 \\ r3-5r1 \ end {array} \ right.$$$ $$$ \ rightarrow \ begin {pmatrix} 1 & 1 & -1 & | & 1 \\ 0 & -1 & 4 & | & -2 \\ 0 & -2 & 9 & | & -3 \ end {pmatrix} \ rightarrow r3-2r2 \ rightarrow \ begin {pmatrix} 1 & 1 & -1 & | & 1 \\ 0 & -1 & 4 & | & -2 \\ 0 & 0 & 1 & | & 1 \ end {pmatrix} $$$ Эта система уже является эшелонированной, поэтому мы можем ее решить. Так мы получили следующую систему: $$$ \ left \ {\ begin {array} {c} x + y-z = 1 \\ -y + 4z = -2 \\ z = 1 \ end {array} \ right.$$$ Решение такое: $$$ z = 1 \\ y = 6 \\ x = -4 $$$ Следовательно, это совместимая детерминированная система.

А теперь изучим систему $$$ \ left \ {\ begin {array} {c} 2x-5y + 4z + uv = -3 \\ x-2y + z-u + v = 5 \\ x-4y + 6z + 2 + v = 10 \ end {array} \ right. $$$

Это мы переписываем как $$$ \ begin {pmatrix} 2 & -5 & 4 & 1 & -1 & | & -3 \\ 1 & -2 & 1 & -1 & 1 & | & 5 \\ 1 & -4 & 6 & 2 & 1 & | & 10 \ end {pmatrix} $$$

Делаем следующие шаги $$$ \ begin {pmatrix} 2 & -5 & 4 & 1 & -1 & | & -3 \\ 1 & -2 & 1 & -1 & 1 & | & 5 \\ 1 & -4 & 6 & 2 & 1 & | & 10 \ end {pmatrix} \ rightarrow (r2 \ leftrightarrow r1) \ rightarrow $$$ $$$ \ begin {pmatrix} 1 & -2 & 1 & -1 & 1 & | & 5 \\ 2 & -5 & 4 & 1 & -1 & | & -3 \\ 1 & -4 & 6 & 2 & 1 & | & 10 \ end {pmatrix} \ rightarrow \ left \ {\ begin {array} {c} r2-2r1 \\ r3-r1 \ end {array} \ right.\ rightarrow $$$ $$$ \ begin {pmatrix} 1 & -2 & 1 & -1 & 1 & | & 5 \\ 2 & -5 & 4 & 1 & -1 & | & -3 \\ 1 & -4 & 6 & 2 & 1 & | & 10 \ end {pmatrix} \ rightarrow r3-2r2 \ rightarrow $$$ $$$ \ rightarrow \ begin {pmatrix} 1 & -2 & 1 & -1 & 1 & | & 5 \\ 0 & -1 & 2 & 3 & -3 & | & -13 \\ 0 & 0 & 1 & -3 & 6 & | & 31 \ end {pmatrix} $$$

и получаем $$ z-3u + 6v = 31 $$.

В этом случае нам нужно присвоить любое значение $$ u $$ и $$ v $$, а затем мы найдем соответствующие значения $$ z $$, $$ y $$ и $$ x $$, все из их согласно $$ u $$ и $$ v $$.Следовательно, это совместимая неопределенная система.

Наконец, давайте посмотрим на пример неопределенной системы $$$ \ left \ {\ begin {array} {c} x + yz = 1 \\ 3x + 2y + z = 1 \\ 5x + 3y + 4z = 2 \\ -2x-y + 5z = 6 \ end {массив} \ right. $$$ Это мы переписываем: $$$ \ begin {pmatrix} 1 & 1 & -1 & | 1 \\ 3 & 2 & 1 & | 1 \\ 5 & 3 & 4 & | 2 \\ -2 & -1 & 5 & | 6 \ конец {pmatrix} $$$ И выполняются следующие шаги: $$$ \ begin {pmatrix} 1 & 1 & -1 & | 1 \\ 3 & 2 & 1 & | 1 \\ 5 & 3 & 4 & | 2 \\ -2 & -1 & 5 & | 6 \ конец {pmatrix} \ rightarrow \ left \ {\ begin {array} {c} r2-3r1 \\ r3-5r1 \\ r4 + 2r1 \ end {array} \ right.\ rightarrow \ begin {pmatrix} 1 & 1 & -1 & | & 1 \\ 0 & -1 & 4 & | & -2 \\ 0 & -2 & 9 & | & -3 \\ 0 & 1 & 3 & | & 8 \ end {pmatrix} \ rightarrow $$$ $$$ \ left \ {\ begin {array} {c} r3-2r2 \\ r4 + r2 \ end {array} \ right. \ rightarrow \ begin {pmatrix} 1 & 1 & -1 & | & 1 \\ 0 & -1 & 4 & | & -2 \\ 0 & 0 & 1 & | & 1 \\ 0 & 0 & 7 & | & 6 \ end {pmatrix} \ rightarrow r4-7r3 \ rightarrow $$$ $$$ \ begin {pmatrix} 1 & 1 & -1 & | & 1 \\ 0 & -1 & 4 & | & -2 \\ 0 & 0 & 1 & | & 1 \\ 0 & 0 & 0 & | & -1 \ end {pmatrix} $$$

Чтобы увидеть несовместимость: $$ 0 = -1 $$.

Эта система несовместима.

Гомогенные системы

Если система с $$ m $$ уравнениями и $$ n $$ имеет все постоянные члены, равные нулю, то говорят, что она однородна.

Он допускает только тривиальное решение $$$ x_1 = x_2 = \ ldots = x_n = 0 $$$

Необходимым и достаточным условием того, что однородная система имеет решения, отличные от тривиального, является то, что ранг матрицы коэффициентов ниже, чем количество неизвестных, или, иначе говоря, что определитель матрицы коэффициентов равен нулю.Следовательно, чтобы решить однородную систему, мы должны сделать ее так, чтобы определитель не был равен нулю, чтобы гарантировать, что ее решение не является тривиальным.

Исключение Гаусса: метод и примеры – видео и стенограмма урока

Что такое метод исключения по Гауссу?

Возможно, вам интересно узнать об этом втором шаге. Что теперь по Гауссу? Исключение Гаусса – это процесс использования допустимых строковых операций над матрицей до тех пор, пока она не перейдет в сокращенную форму эшелона строк.Есть три типа допустимых операций со строками , которые могут выполняться с матрицей.

  • OP1 – Поменять местами две строки.
  • OP2 – Умножить все записи строки на ненулевое число.
  • OP3 – Добавить строку, кратную одной, к целевой строке. (Примечание: целевая строка – единственная строка, которая изменяется в этом процессе.)

Важно понимать, что это всего лишь правила игры. То, как мы будем применять их в той или иной ситуации, будет зависеть от того, какая матрица нам дана.Имейте в виду, что наша цель – преобразовать матрицу в более простую форму, называемую сокращенной формой эшелона строк (RREF) , используя серию этих операций со строками.

Форма сокращенного эшелона строк

Мы говорим, что матрица находится в форме сокращенного эшелона строк, если она удовлетворяет следующим требованиям:

  • При чтении слева направо первая ненулевая запись в любой строке равна 1. Это называется ведущая запись в строке.
  • Первая запись в строке всегда находится справа от ведущих записей в строках над ней.
  • Любой столбец с ведущей записью имеет нули над и под ним.

Вот пример матрицы в форме RREF (не относящейся к нашему примеру). Первые записи выделены жирным шрифтом.

Пример исключения Гаусса

Теперь, когда мы знаем правила игры (операции со строками) и цель (RREF), пришло время разработать пример. Предположим, вы знаете, как найти расширенную матрицу только что рассмотренного примера матрицы.

В первой позиции строки 1 уже стоит 1. Нам нужны нули под ней. Используйте операции OP3. Далее мы используем R 1 для строки 1, R 2 для строки 2 и R 3 для строки 3.

Добавить (-3) R 1 до R 2. Почему следует выбрать именно эту операцию? Дело в том, что R 1 уже имеет 1 на лидирующей позиции. Таким образом, мы можем умножить это на противоположность ведущей записи целевой строки R 2.Когда строки добавляются, -3 отменяет 3, чтобы получить 0 в результате.

(-3) R 1 -3-6 -3-9
+ R 2 3 2 1 3
Результат 0 -4-2-6

Результат заменяет R 2, но R 1 фактически не изменяется в самой матрице.

В первой записи строки 2 стоит -4. Чтобы вместо этого получить 1, умножьте всю строку на (-1/4). Это OP2.

Теперь мы снова используем OP3, чтобы сделать все остальные записи 0 в том же столбце.

Другой OP2 изменит -4 на 1 в ведущей записи строки 3.

Наконец, используйте OP3, чтобы избавиться от ненулевой записи над первой записью в столбце 3.

Прошло некоторое время, но теперь мы поместили матрицу в RREF! Кстати, теперь, когда этапы исключения Гаусса выполнены, мы можем считать решение исходной системы уравнений. Решение находится в последнем столбце: (0, 2, -1).

Краткое содержание урока

Часть процесса решения системы линейных уравнений заключается в использовании исключения Гаусса. Исключение Гаусса – это процесс использования допустимых строковых операций над матрицей до тех пор, пока она не перейдет в сокращенную форму эшелона строк. Этот метод включает выбор серии допустимых операций со строками, которые преобразуют данную матрицу в гораздо более простую форму. Три операции со строками , используемые для решения системы, следующие:

  • OP1 – Поменять местами две строки
  • OP2 – Умножить все записи строки на ненулевое число
  • OP3 – Добавить строку, кратную одной, к целевой строке

Более простая форма называется сокращенной формой эшелона строк (RREF) , в которой:

  • Первая ненулевая запись в любой строке равна 1.
  • Первая запись в строке всегда находится справа от ведущих записей в строках над ней.
  • Любой столбец с ведущей записью имеет нули над и под ним.

Метод исключения Гаусса | Суперпроф

Метод исключения Гаусса – еще один метод поиска решения системы. Он выполняется на расширенной матрице, и мы используем операции со строками, чтобы найти решение конкретной системы. Система должна содержать линейные уравнения, иначе метод исключения Гаусса будет пустой тратой времени и усилий.Другое название этого метода – «сокращение строк». В этом методе есть два этапа: одно – прямое исключение, а другое – обратная замена.

Оба метода разные. Многие студенты думают, что они различаются по операциям, но это неверно, они различаются по результатам. Это означает, что они дают разные результаты. Прямое исключение фокусируется на сокращении строк для перевода расширенной матрицы в эшелонированную форму. Самая большая цель прямого исключения – выяснить, есть ли у системы решения или нет? В случае, если в системе нет решения, значит, нет причин сокращать матрицу на следующем этапе.

Однако, если система выглядит многообещающей, выполняется обратная подстановка, чтобы найти результат решения. Это также последний шаг метода исключения Гаусса, и он найдет результат матрицы.

Система трех уравнений с тремя неизвестными

Метод Гаусса заключается в использовании метода исключения , так что в каждом уравнении на одно неизвестное меньше, чем в предыдущем уравнении.

1.Поместите уравнение с коэффициентом x : 1 или −1 в качестве первого уравнения. Если это невозможно с x, выполните y или z и измените порядок неизвестных:

2. Выполните метод исключения с 1-м и 2-м уравнениями на исключить член x во втором уравнении . Затем во втором уравнении поместите результат операции:

После сложения обоих уравнений:

3.Сделайте то же самое с 1-м и 3-м уравнениями с по исключите член x :

После сложения обоих уравнений:

4. Выполните метод исключения для 2-го и 3-го уравнение:

Сложение обоих уравнений:

5. Получена другая эквивалентная система:

6. Решите систему:

Примеры

Q.1

Добавление обоих уравнений:

Добавление обоих уравнений:

Добавление обоих уравнений:

Q.2

Сложение обоих уравнений:

Добавление обоих уравнений:

В результате сложения обоих уравнений:

9001 4

Лучшие репетиторы по математике

Первый урок бесплатно

Задачи со словом

Q.1 Покупатель в супермаркете заплатил в общей сложности 156 долларов за 24 литра молока, 6 кг ветчины и 12 литров оливкового масла. Подсчитайте цену каждой позиции, зная, что 1 литр масла стоит в три раза дороже 1 литра молока, а 1 кг ветчины стоит столько же, сколько 4 литра масла и 4 литра молока.

молоко x

ветчина y

оливковое масло z

Размещение

:

молоко 1

долларов

ветчина 16 долларов

оливковое масло 3 доллара

Q.2 Видеомагазин специализируется на фильмах трех жанров: детский, вестерн и ужасы. Известно, что:

60% детских фильмов плюс 50% вестернов составляют 30% всех фильмов.

20% детских, 60% вестернов и 60% фильмов ужасов составляют половину всех фильмов в видеомагазине.

Вестерн-фильмов на 100 больше, чем детских.

Найдите количество фильмов в каждом жанре.

дети x

западный y

ужас z

Поместите уравнение 3 в оба уравнения:

Сложение обоих уравнений:

дети 500 фильмы

вестерн 600 фильмы

ужасы 900 фильмы

Q.3 Стороны треугольника 26, 28 и 34 см. В центре каждой вершины расположены три касательные друг к другу окружности. Вычислите длину радиуса каждого круга.

15.5 Важные наблюдения относительно исключения Гаусса

15.5 Важные наблюдения относительно исключения Гаусса

15.5 важных замечаний по поводу исключения Гаусса

1. При выполнении элементарных строковых операций исключения Гаусса сделать матрицу коэффициентов в единичную матрицу, вы должны выполнить те же операции одновременно в правой части уравнения, на r в примере.
При этом вы можете написать v как I 3 v , оставив v исправлено и выполните операции со строками на I 3 , как вы их делаете на С .
Когда вы закончите, вы преобразуете I 3 в некоторую новую матрицу, назовите его D .

r = I 3 r = D v

, а у нас также есть

C r = v

D , следовательно, является инверсией C :

D = C -1

Метод исключения Гаусса обеспечивает относительно эффективный способ построения обратная к матрице .

2. Точно такие же результаты справедливы для любого количества переменных и уравнений. Метод исключения Гаусса практичен в большинстве случаев для нахождения обратный к матрицам, включающим тысячи уравнений и переменных.
Однако исключение Гаусса – чрезвычайно скучная повторяющаяся процедура, не очень подходит для людей. К счастью, компьютеры никогда не скучают с выполнением элементарных операций со строками, и их можно легко обучить сделать гауссовское исключение.

3. Исключение Гаусса обеспечивает простой способ оценки определителя матрицы : произведение всех количеств, деленное на уменьшение в строке – величина определителя матрицы. Это сам по себе определитель если в процессе сокращения рядов нет перестановок рядов; иначе каждая простая перестановка двух строк меняет знак.
Таким образом, в приведенном выше примере шаги 1, 3 и 5 включали деление на 2, -5/2 и 1/5 соответственно.Их произведение равно -1, что является определителем этой матрицы.
Это один из самых простых способов вычисления детерминантов в целом.
Пожалуйста, поймите, что способ, которым вы научились оценивать детерминанты: умножать по диагоналям с соответствующими знаками, работает только для массивов 2 на 2 или 3 на 3. Это абсолютно неверно для более крупных детерминант.
Интересный вопрос: как быстрее всего вычислить детерминанты? и инверсии и произведения матриц.Метод исключения Гаусса принимает порядок из n 3 операций для матрицы n на n; брать продукты в очевидном путь занимает в таком же порядке времени. Есть и другие умные способы сделать это вещи, которые занимают порядка n 5/2 шагов для очень большого n, но они не очень полезны на практике, так как им не хватает числовой стабильности, так что маленькие ошибки могут ужасно разрастаться; также они не поддаются параллельные вычисления, а также стандартные.

Упражнение 1.5: Матрица: метод исключения Гаусса

УПРАЖНЕНИЕ 1.5

1. Решите следующие системы линейных уравнений методом исключения Гаусса:

(i) 2 x – 2 y + 3 z = 2, x + 2 y г = 3, 3 x y + 2 z = 1

(ii) 2 x + 4 y + 6 z = 22, 3 x + 8 y + 5 z = 27, – x + y + 2 z = 2



2.Если топор 2 + bx + c есть разделить на x + 3, x -5 и x -1, остатки 21, 61 и 9 соответственно. Найдите a , b и c . (Используйте метод исключения Гаусса.)



3. Сумма в размере 65 000 фунтов стерлингов вложена в три облигации на ставки 6%, 8% и 10% годовых соответственно.Общий годовой доход составляет 4800 драм. Доход от третьего облигация на 600 фунтов больше, чем вторая облигация. Определите цену каждого связь. (Используйте метод исключения Гаусса.)



4. Мальчик идет по тропе y = ax 2 + bx + c через точки (-6, 8), (-2, -12) и (3, 8). Он хочет встретиться со своим другом по телефону P (7, 60). Он встретит своего друга? (Используйте метод исключения Гаусса.)



Ответы к упражнению 1.5:

1. (i) x = -1, y = 4, z = 4

(ii) x = 3, y = 1, z = 2

2. a = 2, b = 1, c = 6

3. ₹ 30000, ₹ 15000, ₹ 20000

4. a = 1, b = 3, c = -10, да

Теги: Проблемные вопросы с ответом, решение, 12-е место по математике: приложения матриц и определителей

Учебные материалы, лекционные заметки, задания, ссылки, объяснение описания вики, краткая информация

Математика 12-е: приложения матриц и определителей: Упражнение 1.5: Матрица: метод исключения Гаусса | Проблемные вопросы с ответом, решение

.

Оставить комментарий